Vous êtes sur la page 1sur 99

Manu Recalls

04/12/2014

Do recalls very well they are time saving and you will see them in your
exam for sure
Please pray for me

1-Chardiovascular risk calculation chart from jm


1- Pt with MI 5 years a go come with total cholesterol;4.5
2- Pt with bp;140/95 bmi;28 smoker fbs;5.4 stroke in father at the age of 70
mx? Study tables in jm chapter of HTN
3- GDM screening
4- A 41 y/o female flushed with artheralgia weight loss night sweeting t; 37.4
which reach 38 most appropriate next step?
Ana
Bc
Ds dna
Ans : If the full stem denotes SLE then A
If not then B
5- Male with homosexual partner watns hiv screening after some weeks
come with fever sore throught exudate hepatosplanomegali and
macolopapular rash and the result of elisa and westernblot were negative
what will you do?
Repeat hiv tests
Ebv test
Ans: Ebv test

JM252

6- Mm scenario
7- Pt with cough breastlesness and an xray which a balck mass was seen in
cxr and there was h/o smoking I choose lung cancer
Ans: mostly that is right Scc
8- Pt with hypercalcemia phosphor upper limit of normal with rise in cr and
urea dx?
Renal failure
Malignancy
Ans: mostly Malignancy as in CRF there is hypocalcemia
Adam Jp It looks like Malignancy (MM) bc can cause renal failure and hypercalcemia Ireen so I think B
bc renal failure due to other hypocalcemia not hyper. here hypercalcemia is due to Malignacy not renal
failure I

9- 2 qustion about collangitis scenario with hypotention and ercp


10-A girl with in intermittent pain in lower abdomen for 3 days which worse
during micturition at 9 days after ending of her period and has such pain
in 4 weeks ago which spontanusly disappeared after 3 days dx?

Ovulation
Ep
Ans: Ovulation..but do not forget endometriosis if the pain is of
long period
11-Placenta abruption
12-Fat female with oligomenorea fsh2.2 lh8 prl470 tsh8 next sep?
Brain mri
Vaginal us
Progestron
Testestrone
Ans: A controversy
Shilpi Singh if TSH and PRL under normal range then testosterone to confirm PCOS. but if tsh n prol
increased then A

13-As naproxen
14-Dermatophibroma picture
15-Infant with drowsiness poorfeeding ill fever rash what to do
Penicillin
Bc
Ans: Penicilin JM271
16-Ctpa
17-v/q scan
18-tt vaxination
19-hepatitis A or malaria
20-needle stick hbsag positive next? Concelling
21-mulerian agenesia
22-terner
23-12 month old infant cannt walk alone but other milstone were normal what
to do?
Review 3 month later
24-Old female after husband die with depretion symptom
25-Party drug abuse
26-Old guy with police say maphia control me
27-Psvt accompanied wpw ecg?
Sotalol
28-Palpitation and lightheadness after running for 15 kms which ecg?
Multiple pvc
Af
Atrial flutter
V fibr
Ans: the answer is not included as the answer her has to be SVT or less
likely VT
29-Dig toxicity ecg
30-Hirshprong scenario next step
Xray
Barium flow through
Ans: xray (to be reviewed after studying the subject from JM)
31-Giardia symptom in truk driver

32-Nursing home resident pt with dearrhea for 4 days and fecal incontinency
next step
s/e s/c
sigmoidoscopy
colonoscopy
Ans: colonoscopy
Nikita Bhatia jm pg number 319..... fecal impaction is encountered typically in the aged, bedridden
debilitated patient. it may closely resemble malignant obstruction in its clinical presentation. spurious
diarrheo can occur which is also known as fecal incontinence
Nikita Bhatia so in order to rule out malignancy we will do colonoscopy...
Nikita Bhatia according to kaplan... colonoscopy is clearly the most accurate diagnostic test....
sigmoidoscopy will only reach the lesion within the distal 60 cm of colon. If the lesion is there then the
sensitivity of sigmoidoscopy is equal to colonoscopy. only 60% of cancers occur in distal area

33-Some cirrhosis symptom with fever abdominal pain and confusion dx?
Sbp spontaneous bacterial peritonitis
34-LSIL
35-EDH ct scan
36-Headache pecialy after coughing for 3 weeks after mva dx?
EDH
Subdoral heamatoma
Post tromatic syndrome
Brain concoction
Ans: Subdural haematoma
Fady Nasif B ? it must have a vascular component as long as it worsens with cough
Fady Nasif B can be presented by subacute type and it's d.t venous leaks/tears rather than arterial tears
with EDH , so cough inc the headache ?
Mahwish Sheikh in EDH there would be luicid interval ,severe headache and other neurological signs
also.

37-Sever croup with syanosis first step?


O2
38-Gord scenario
39-Renal failure with unilateral artery stenosis
Ace i
40-50 y/o man wants prostat cancer screening but doesnt have any risk
factor or symptom
Talk about pons n cons of prostate screening
Psa n dre
41-Domestic violence in female dont want any body know
42-Aboriginal mom without eye contact but there wasnt cultural issue so I
choose bring one of family member with you in next visit
43-Postnatal depression beside treatment what to do
Family involve
44-Schisophreni treatmat
45-Old pt with abdominal distention constipation n use lagxative for long time
dx?
Sigmoid volvulus

46-Secom caner symptom


Lethargy
47-Pt on pneumonia treatment with penicillin n azithromycin after 2 days rash
appear what to do? There was pic of vesicopostular rash
Allergy drug test
Take swab from lesions
Ans: B Mostly

But if steven Johnson syndrome the answer is A

Noor Eldeen whats the dx ?? may be steven johnson syndrome !! Michael Gamal
Noor Eldeen steven johnson syndrome is also severe reacton may cause dt drug allergy
Noor Eldeen SJS may be caused by adverse effects of the drugs vancomycin, allopurinol, valproate,
levofloxacin, diclofenac, etravirine, isotretinoin, fluconazole,[11] valdecoxib, sitagliptin, oseltamivir,
penicillins, barbiturates, sulfonamides, phenytoin, azithromycin, oxcarbazepine, zonisamide, modafinil,
[12] lamotrigine, nevirapine, pyrimethamine, ibuprofen,[13] ethosuximide, carbamazepine, bupropion,
telaprevir,[14][15] and nystatin.[16][
Noor Eldeen yes i think if dx is sjs then a
Alain Diago SJS is a spreading purplish/reddish skin rash that spread. And it may have blister. And
shedding of skin. But here its vesicopustular rash I think something wrong with the description of the
Skin rash. .

48-64 y/o Pt with cough sputum fever cracle and widsprede patchy opacity in
cxr treatment?
Penicillin
Coamoxyclave
Doxyciclin
Ans: needs work out on pneumonia, but If atypical pneumonia then
Doxycicline JM 269 otherwise coamoxclave is the answer
49-Old female with gonorhea without sexual contact what to do
Cefteriaxon iv
50-Cavazaki mx
51-Symptomatic uti in 13 y/o girl
52-Asthma dx
53-Bowel obstruction symptom with a lump below n latral pubic tubercle ix?
Ct
Xray
Us
Fna
Ans: xray
54-Lump in below n latral pubic tubercle which is not coming out with cogh or
reduced ix?
Fna
Ans: fna is wrong answer the best answer may be sonar
55-How dx incisional hernia?
Lye down n cough
56-Gram negative septisemia
57-Acne and weight gain is side effect of?
Li
Respridon
Aggmentation of li n respridone

Ans: Lithium
58-Diverticulitis senarion next step?
u/a
us
ct
59-Asperger
60-Most common palce of endometriosis
Uterosacral ligament
61-Child with headache
Paracetamol
62-Girl with mam wants clamidia screening mom rings for result
Bring your doughter
63-Alcohol and concive
64-Preeclampsia dx
65-Adolescent with depretion n insomnia
Venlafaxine
66-Breast cyst ix
67-Breast solide cyst whats the type?
Dermoid cyst
68-Diabetic foot ulcer with yellow discharge what to do?
Mri
69-Sudanian kid with sezure less than 1 mine without fever is verywell right
now what to do?
Ct
Eeg
Vit D level
Ans: mostly EEG Jm 1259..but some suggested Vit D level
Aryan Saif Mujtaba I think it is C...because I read a article fromhttp://www.ncbi.nlm.nih.gov/pubmed/
"There is growing interest concerning the role of vitamin D in various medical conditions such as
diabetes and oncological, cardiovascular and central nervous system disorders. Although vitamin D
deficiency is known to be highly prevalent among epilepsy patients, only a single study, published nearly
forty years ago, assessed the effect of vitamin D on seizure control. Here, we measured serum 25hydroxy-vitamin D (25(OH)D) levels and normalized it by administration of vitamin D3 in 13 patients with
pharmacoresistant epilepsy. To see if vitamin D3 has an impact on seizure frequency, we compared
seizure numbers during a 90-day period before and after treatment onset. We found that seizure
numbers significantly decreased upon vitamin D3 supplementation. Median seizure reduction was 40%.
We conclude that the normalization of serum vitamin 25(OH)D level has an anticonvulsant effect"
Hina Asher Eeg will be inconclusive , it's sudane, the most common group for vit d deficiency because
of dark skin that is resistance to uv light so they are always deficient to vit d that will impair ca absorption
as vit d is needed for calcium so hypcalcemia is the cause of these seizures correct vit d level and
reassure
Emad Askary Mary Melad, B. See JM p.1259, standard minimum investigations.

70-Epigastric pain radiates to the back settle after 2 hours n pt is fine what to
do?
Lipase
Gasteroscopy
Ans: Lipase (acute pancreatitis)
71-Pancreatic cancer has worst prognosis
72-Pancreatic caner scenario ix?
Ct

73-Thin membrain disease recall


74-Ortostatic proteinuria dx n ix
75-HOCM scenario ix
Echo
76-Keratitis
77-Dependant personality
78-Borderline or conduct recall
79-Pt is confused and his layer want you witness the will
80-Sle pt on nsaid beside that what will you give
Hydroxycolorquine
81-Oxycodone urine level recall
82-Kid with pertosis what will you do?
Ab and exclusion
83-6 y/o child with hematuria and proteinuria dx
Iganephropathy
Psgn
ATN
They didnt say any thing about history if any infection
Ans: as long as there is no history of infection we have to think of IGA
neohropathy
If infection present then PSGN
84-Liver abcess
85-Child with petechi n easybruising cervical lanphadenopathy plt=5000
What to do?
Plt
Chemotherapy
Ivig
Ans: if the complete scenario contains history of viral infection then it is
ITP and IvIg is the choice while if the full stem shows that all other blood
series are affected then it is ALL and chemotherapy is the best choice
86-Pathological fx in spain with its xray what to do?
Bone scan
Serum protein electrophoresis
Psa
Ans: mostly Bone Scan..but think about DEXA scan if you suspect
osteoporosis and Psa if you suspect malignancy
87-Femal with non productive cough for 3 days otherwise very well what to
do?
Come back if not improve
88-OA and sticker
All the best
Please pray for me
Amc-MCQ 14 oct 14
1) Case on a child with slight jaundice on 2 nd day of birth. Cause?

2) 18 yr old athlete, father died at the age of 40. What investigation? Echo?
3) Defence mechanisms ; altruism
4) Girl with OCd, excessive hand washing. c/o Rash onn hands due to
compulsive handwashing treatment? Sorbolene cream, fluvoxamine ,
olanzapine
5) Child with svt (no ecg) how do you fix it? Cold water was an option
6) A nursing home bus driver had a grand mal epilepsy with no trigger
factors. What will you advise him? A) drive after 6 months B) never drive
C) report to the authorities
7) Lots of psychiatry
8) Preg woman wth unsymptomatic ovarian cyst, what to do?
9) Mucinous cystademona
10)
Question on capsule endoscopy (old recall)
11)
Herpes on the trunk, asking for treatment.
12)
Swelling on the anterior 1/3 of sternocleidomastoid. Diagnosis?
Brachial cyst
13)
19 year old, fights with everyone, keeps stealing stuff. Diagnosis?
Antisocial personality
14)
Woman with normal hb, wcc but low platelet counts (35) what will
you do? A) give platelets and other random useless options
15)
Recurrent uti, bilateral swelling,sister had the same problem a)
horseshoe kidney b) cant remember other options
16)
Patient with neck stiffness only a) meningitis and other options
17)
Malingering case from the old recall.
18)
Hypnogogic hallucination?
19)
An xray photo of tibia and fibula, the tibia is completely displaced.
Asking for management? A)external fixation b)internal fixation
I only got one Ecg, but lots and lots of psychiatry.

Thank you everyone for all your support and help. Please remember me in
your prayers. May we all pass. Confidence is key! And ofcourse a good nights
sleep!
(Ps., sorry for the delay I did not have internet access earlier, and this is all
that I can recall, I hope this helps)

1. Man with history of alcoholism has inability to flex fingers and


grasp objects. Which of the following will you do before intervention?
Pic similar to this one but ring finger flexed.

A) X-ray hand
B) MRI
C) USG Hand
D) CT
Ans:C
Causes of the disease are mostly genetic as the disease runs in families
and it is more in men than in women and it is seen most often in people of
Northern European (English, Scottish, Irish, Dutch, French) or Scandinavian.
Shilpi Singh No routine radiographs are necessary, but ultrasonography can demonstrate thickening of the
palmar fascia, as well as the presence of a nodule. In addition, ultrasonography of a thickened cord may be
useful prior to intralesional injections so that the underlying tendon can be identified and avoided during the
injection.

2. Man comes with complains of pain in right ear. He also complains


of blood mixed with saliva. Which of the following if affected will lead to
this condition?
A) Gums
B) Larynx
C) Tongue
D)Trachea
Ans: c..tongue carcinoma

3. Man with history of hemochromatosis. He is in intensive care


unit. Has flapping tremors. He is confused and restless. He also has
ascites with shifting dullness. Ascites tap done which shows

Cells 300
LDH
Which of the following is the most appropriate immediate
management?
A) Amlodipine
B) Albumin
C) Lactulose
D) Cefotaxime
Ans: D
Mahi Shah Fari Shaikh agreed that it's hepatic encephalitis and lactulose would definitely be included in
management .....
But see what they have asked is immediate management....
Lactulose is basically non-digestible sugar prevent ammonia absorption. Thus helpful in prevention of further
development of encephalitis but NOT CURE OF IT.
Spontaneous bacterial peritonitis is LIFE THREATENING infection of ascitic fluid, furthermore antibiotic will also
prevent ammonia production by reducing bacteria in gut......
I hope this helps, I tried to find reference for you but information is scattered...
Mahi Shah If the fluid contains bacteria or large numbers of neutrophil granulocytes (>250 cells/L) (a type of
white blood cells), infection is confirmed and antibiotics are required to avoid complications. In addition to
antibiotics, infusions of albumin are usually administered.
NOTE: In our question cells are 300......
http://en.m.wikipedia.org/.../Spontaneous_bacterial...
Mahi Shah Fari Shaikh these are recall questions so might be possible that person who recalled qs skipped fever
part...... Even if fever is not given, do you think we should wait for fever to occur in order to commence antibiotics
in life threatening condition ?

4. Man who is mechanic and drives commercial vehicle has history


of seizures. He has been advised to not drive for the next 6 months. He
however believes that he should continue driving as he doesn't want to
disappoint his customers. He starts driving within 2 weeks of having
seizure. Which of the following is the most appropriate action?
A) Notify his employer
B) Notify licencing authority
C) Let him drive as he needs to continue work
D) Ban him from driving
Ans:B

Shan Saleheen B, DLA will make the decision, but, treating doctor can give a non-judgmental medical report to
DLA. "Expectations of patients and driver licensing authorities (DLAs) that treating doctors should adjudicate on
fitness to drive undermine the doctorpatient relationship, discourage reporting of seizures, encourage doctorshopping and expose the practitioner to legal risk.19 The authors preference is to provide a concise medical
report allowing the DLA to make the decision (Figure 1)."
Shan Saleheen http://www.racgp.org.au/afp/2014/march/epilepsy/

5. Mother presented with 2 year old girl child who has genital warts.
Mother has history of genital warts during pregnancy which has resolved.
Which of the following is the most likely cause?
A) Auto inoculation by the mother
B) Child acquired it during birth
C) Sexual abuse
D) Acquired infection from water source
Ans: c
Alexander Kuzminov <2 years - B, more than 2 years C
Eman Khamis Suspect sexual abuse if wart is in child jm

6. Somalia woman came to your GP clinic with child who as


vaccinated with prior OPV vaccines. You have only 2 doses of IPV vaccine
left in your practice. What to you do?
A) Give IPV
B) Order OPV and give vaccine after 2 weeks
C) Don't continue vaccination
Ans: A
7. Parent presented with child who they found wandering around
the house in the morning. He as confused and dint respond very well.
Now they are worried. Which of the following will aid in your diagnosis?
A) Serum Creatinine, Urea and Electrolytes
B) Blood Glucose
C) Serum Iron
D) Urine Analysis
Ans:B
Emad Askary Shona Amber, B. JM p.803: Hypoglycaemia usually has preliminary symptoms of hunger,
sweating, shaking or ALTERED BEHAVIOUR. The cause of hypoglycaemia in this child can be fasting
hypoglycaemia which is classically seen between 18 months and 5 years. (See the website of The Royal
Children`s Hospital Melbourne for more details)

8. There was a question about CTG. It hit me like a bolt of


lightening. The stem said match the following correctly. The 5 options
were huge and was like 75% of the screen space. It took about 2 minutes
to read it and then to process the information took more time. All the
options put together was basically looking for relations between
accelerations, decelerations, CTG pattern and cause. One was vasa
praevia. So study that.

9. ECG of Torsade de Pointes was given. Boy was started on


sertraline for depression. Later he was started on erythromycin for some
infection. He now presents to Emergency Department after collapse.
Which of the following s the cause of the ECG?
A) Sertraline
B) Sertraline and Erythromycin
C) Drug abuse

Ans : erythromycin causes torsade de points while sertraline not. So if


erythromycin is present alone then it is the right answer. Otherwise B
NB: there is no interaction between sertraline and erythromycin

10. Old lady is brought to Emergency after a MVA. She is has been
the blood pressure was 100/ 80 and now it is 90/50. Which of the
following is the most likely cause of her status.
X-ray was given. The X-ray was very similar to this one.

A) Pneumothorax
B) Haemothorax
C) Pulmonary contusion
D)Ruptured Thoracic Aorta
Ans :d
Meshkatul Islam Hb 3.070..same xray..widened mediastinum..its D n an incomplete recall..

11. 75 year old man come with complain of right upper quadrant
pain. He has mild jaundice. USG was done which showed no stone in the
bile duct. However the duct is dilated. Which of the following is the most
appropriate management?
A) MRCP
B) Cholecystectomy
C) ERCP

D)X-ray
Ans: C

12. Child comes to you with complains of pain in the right knee. You
notice that he is limping while walking. He has mild temperature. On
examination you notice that there is decrease movements in the right hip.
USG shows that there in there is widening of joint space. Which of the
following it the most appropriate management?
A) Antibiotics
B) Analgesics
C) Immobilise
D) Steroids
Ans:b

Mahwish Sheikh Irritable hip (or transient synovitis) is the most common cause of limp in children. It is due to
inflammation of the lining of the hip joint. The cause is unknown, but in some cases your child may have had a
recent viral infection. Sometimes this condition occurs after a fall or injury. It is most often seen in children aged
between three and 10.
Mahwish Sheikh X-rays of transient synovitis of the hip are typically normal; however, they may reveal subtle
signs early in the disease process such as capsular distension, joint space widening, diminution of the definition
of soft tissue planes around the hip joint, or slight de-mineralisation of the bone of the proximal femur.
Rumaisa Shaukat B here . But septic arhritis also cancause widening of joint space but temp willbe higher and
restriction of movements kid refuses to walk

13) Man comes to presents to Emergency Department and he is


very violent. The security guards are trying to hold him down with great
difficulty. Which of the following will be you choice of management.

A) Try to pacify him verbally


B) IV Diazepam
C) IM Haloperidol
D) Thiamine
Ans: mostly c

14) There was pic of eye with which showed conjunctivitis. Stem
said girl complained of discharge and pain in the eye with redness. She
also complained of mild back pain. (The pic nor the history dint have any
features of Uveitis. Also no history of AS). Which of the following in the
most likely cause?
A) Allergic
B) Bacterial
C) Viral
Ans: May be b (we have to wait to see the full stem)
Rumaisa Shaukat Why back pain in conjunctivitis
Rumaisa Shaukat Episcleritis here
Oveem Istanis Rumaisa Shaukat Episcleritis has no discharge???!!!!
JM 546
Oveem Istanis According to JM, Allergic has to be bilateral
bacterial bilateral and prulent discharge
viral uni/bilateral and watery discharge
so it depends on the question and the pic
JM 543
Namrah Shafiq may be anterior uveitis as pain tearing redness are the features ass with as as back pain in the
hx
Ireen Baker Ahmed If discharge watery it could be a or c Ahmad
Eman Khamis B not a because no history of atopy not c because there is redness disharge and pain

15) In a particular population 8 people have prostate cancer out of


100. The new drug reduces the prostate cancer by 25%. How many
people have to be treated?
A) 100
B) 200
C) 50
D) 3
Ans: c
Dr-Sarmad Sameer oc8.15 c 25% i.e 1/4*8=2 1/2*100=50

16. Child was brought to Emergency Department after consuming


some, white tablets which belonged to grandmother. Grandmother has
been started on treatment for herpetic neuralgia. Which of the following
would be the most likely presentation of the child?
A) Respiratory depression
B) Coma
C) Delirium
D) Dehydration
Ans: b
It is TCA overdose

http://www.rch.org.au/.../guideline.../Tricyclic_Overdose

17. Aboriginal woman comes to you feeling depressed following the


death of her husband. She says that she knows spirits are responsible for
the death of her husband. She also says that she suspects that her
relatives many have involved spirits in her husbands death. How will you
manage?
A) Prescribe antipsychotics
B) Admit her
C) Manage her with the help of aboriginal social worker.
D) Talk to family
Ans: c

18. Old lady presents to the Emergency Department with abdominal


dissention and pain. She has no complains of vomiting or diarrhoea. She
has history of appendectomy which was 20 years ago. Which of the
following is the most likely cause of presentations?
A) Adhesions
B) Colon Cancer
C) Fecal Impaction
D) Volvulus
Ans: b
Absence of vomiting excludes sbo as adhesion
Absence of diarrhea excludes faecal impaction
The commonest cause for lbo is colon cancer

19. 25 year old pregnant lady with family history of DM comes to


you during the second trimester of pregnancy. Which of the following in
the most appropriate?
A) OGTT 26-28 weeks
B) OGCT 26-28 weeks
C) FBS
D) OGCT now
Ans: D
JM 1016
OGCT if abnormal OGTT
20. A lady who is pregnant come to your GP clinic at 8 weeks. She
has history of postpartum depression. She was treated during her
previous pregnancy for sever depression. What will you do regarding
depression in the current pregnancy?
A) Start her on treatment now
B) Review her later in pregnancy
C) Start prophylactic treatment after delivery
D) Review her after pregnancy
Ans: B

21. A young female comes to you with history of criminal offences.


She has had problems with the police and has been in prison. She has
history of drug abuse. Which of the following is the most likely diagnosis?
A) Borderline Personality Disorder
B) Antisocial
C) Conduct disorder
D)Substance abuse
Ans: b

22. A 48 year old man come to you with history of back pain which
started after lifting heavy objects. Pain decreased after about one week.
No he develops pain in the back with tenderness around the L2 region. He
also has a temperature of 37 degrees (I think). What is the next step in
management?
A) CT Scan

B) MRI Spine
C) X-ray
D) Bone scan
Ans: mostly b.As

Plain X-rays of the lumbar spine are not routinely

recommended in acute non-specific low back pain (pain


<6 weeks) in the absence of 'red flags' as they are of
limited diagnostic value and no benefits in physical
function are observed.' J.M P.381

We have to review JM for investigations 381 and 376 for red flags

23. A 60 year old man is brought to you by this wife after he had a
fracture. The wife was diagnosed with osteoporosis a year back following
a fall and fracture. The old mans BMI is only 15 and he has family history
were the BMI has been in the lower end of normal. What will you do next?
A) X-ray
B) CT
C) Bone Scan
D) Serum Calcium
Ans: mostly c
Emad Askary Mahwish Sheikh, DEXA scan. JM5 p.1049: One in three men >60 years of age will suffer an
osteoporotic fracture. JM5 p.991: BMI < 18 is a risk factor for osteoporosis. DEXA scan is the gold standard for
the diagnosis of osteoporosis. I guess the fracture has been appropriately managed and the patient is being
visited during the follow up.

24. A mother brings a child to you. Everyone in the family is


overweight or obese. The childs BMI is only 20. What will you do?
A) Reassure the mother that he is normal for his age
B) Prescribe supplements
C) Estimate bone age
Ans: difficult to be answered because the child age is not provided.but
may be c
Reshma Rafi bmi is not reliable in chidren.. but among these i go for a
Reshma Rafi Here we can counsel the family.. Cannot commence rx now
Alain Diago we need the age of the child here. to compute for BMI should be weight for age. Age will determine if
child is normal, underweight or obese. BMI of 20 for a 7 year old child can be obese and BMI of 20 for a 12 year
old child can be normal

25. Mammography although not mandatory in Australia, is now


offered to women under a Government scheme. It has helped in early
detection of breast cancer in many women. Which of the following is a
concern about mammography screening in Australia?
A) Missed diagnosis of breast cancers
B) It is expensive
C) Mastalgia
Ans :A

JM 951

26. A 50 year old man comes to you with history of reflux and burning sensation
in epigastrium and. He is heavy smoker and alcoholic. An endoscopy was done
as part of his assessment which showed a reddened areas in the lower
esophagus. What is the most appropriate management?

A) Decrease alcohol
B) Quit smoking
C) H Pylori testing
D) PPIs

Ans: D proton pup inhibitors

27. Man comes to you with history of progressive dysphagia. He


initially developed dysphagia for solids. Now he has dysphagia for liquids
as well. During night, he has regurgitation of food. Which of the following
is the most likely cause?
A) GORD
B) Esophageal Cancer
C) Achalasia
D) Barretts esophagus
Ans:
c
cancer B

..But if there is prominent loss of weight it is oesphageal

Emad Askary Shona Amber, C. JM5 p.511: Intermittent dysphagia for both liquids and solids is characteristic of
a motility disorder such as achalasia. Malignant oesophageal obstruction is usually evident when there is a short
history of rapidly progressive dysphagia and SIGNIFICANT WEIGHT LOSS.

28. Young couple comes to you. The husband has history of


Rheumatoid Arthritis. He is on methotrexate and sulfasalazine. They
planning on starting a family. What will you advise?
A) Continue methotrexate stop sulfasalazine
B) Continue both methotrexate and sulfasalazine
C) Stop both methotrexate and sulfasalazine
D) Stop methotrexate continue sulfasalazine
Ans: c

..as sulfasalazine has no effect on semen

29. Man around 30 years comes to you for enquiry about prostate
cancer screening. His father died of prostate cancer at the age of 75. His
uncle also has been detected with prostate cancer. What will be your
advice?
A) He needs screening now
B) Regular 2 yearly screening

C) Nothing to worry and reassure


D) Start screening at aged 40
Ans: mostly c and may be with little possibility d
According to RACGP Screening for prostate cancer is not recommended unless:
1. the man specifically asks for it; and
2. he is fully counselled on the pros and cons
Also see JM 1077
Shan Saleheen 'Screening is not recommended unless the patient specifically requests it and he is fully
counselled on the pros and cons of screening.' RACGP recommendation

30. A woman, 30 years old comes to you with to enquire about


breast cancer screening. Her mother was diagnosed of breast cancer at
the age of 65. Her older sister has been diagnosed of ovarian cancer at
the age of 36. What will be your advice? (Im very sure of the age in the
question)
A) Start mammography now
B) 2 yearly USG
C) Self breast examination
D) Reassure her
Ans: mostly d
Rumaisa Shaukat D here Meshkatul Islam unless Potentially high risk
(<1% of the female population)
Women who are at potentially high risk of ovarian cancer
Two first- or second-degree relatives on one side of the family diagnosed with breast or ovarian cancer plus one
or more of the following features on the same side of the family:additional relative(s) with breast or ovarian
cancerbreast cancer diagnosed before age 40 yearsbilateral breast cancerbreast and ovarian cancer in the same
womanAshkenazi Jewish ancestrybreast cancer in a male relative.
Dr-p Bn D.recommended mammography by age 30 years in women with 20% lifetime risk for breast
cancer based on family history...so in women because we dont have any personal and family history of BRCA
and age is less than 35, we dont need to do mammography....before we going to say she is high risk we need to
do risk assessment ... anyway genetic counselling is best option here . Dr-Sarmad Samee
Shilpi Singh D This is from racgp..mammograms become more effective as women get closer to age 50
years. Mammographic screening is not recommended for women aged <40 years because the reduced accuracy
of mammography produces a high risk of false positive and false negative results.506
Women aged 70 years are eligible for free 2-yearly screening mammograms through BreastScreen Australia,
although they are not targeted by the program..Population-based screening using mammography is the best early
detection method available for reducing deaths from breast cancer.508 Evidence of the benefit is strongest for
women aged 5069 years. Women aged 4049 years are eligible for free 2-yearly screening mammograms
through BreastScreen Australia, although they are not targeted by the program.
See also JM 172

31. A middle aged lady presents with painless mass just below the
pubic symphysis. What is the most appropriate management?

A) Surgery
B) FNAC
C) USG
D) Nothing to be done
Ans: C ..unless it is confirmed to be hernia then surgery(the full stem
will determine)

32. 22 year old man comes to you with family history of colorectal
cancer. Father was diagnosed with CRC at the age of 50. Uncle had CRC.
Elder brother has been diagnosed at the age of 30. What will be your
advice?
A) Colonoscopy now
B) FOBT 2 yearly
C) Reassure
D) Start screening at 40
Ans:A
JM 68

33. A young man was brought to the ED by his friends. He had an


episode of up rolling of eyes. He did not have any history of seizures. He
is quiet embarrassed by the episode. Which of the following in the
appropriate treatment?
A) Haloperidol
B) Olanzapine
C) Benztropine
D) Diazepam
Ans: if acute dystonia then C
Emad Askary Shammi Nasreen, I think the stem is incomplete. I think we need more information to differentiate
among the causes that are presented in JM p.801 table 76.4. Useful information include precipitating factors, drug
history, the presence or absence of postictal confusion.

(JM)

Movement disorders from antipsychotic medication 1


Acute dystonias
usually bizarre muscle spasms affect face, neck, tongue and trunk
oculogyric crises, opisthotonos and laryngeal spasm
Rx: benztropine 1-2 mg IV or IM

Akathisia
subjective motor restlessness of feet and legs
generally later onset in course of treatment
Rx:
reduce dosage until akathisia less troublesome or substitute thioridazine
can use oral propranolol, diazepam or benztropine as short-term measure

34. An elderly male presented to you with an abdominal mass. The mass
increases in size when he lies down and raised his shoulders and trunk. What is
the most appropriate management?
A) Excision
B) Support
C) Herniorrhaphy
Ans: B Divercation of recti it needs physiotherapy

35. 50 year (I think) old woman with no significant past history. She
has history of tubal ligation. No she presents with hot flushes and
Mastalgia. Which of the following is the most appropriate management?
A) HRT
B) Cyclic estrogen progesterone
C) Continuous estrogen progesterone
Ans:b
May need to have a look over the menopause JM983

36. A young female has come to you with painful loss of vision. This
has progressed over the last 12 hours. All examinations are normal
except for visual acuity which is 6/60 and 6/6. What will aid you in
diagnosis?
A) Temporal artery biopsy
B) Lumbar puncture
C)MRI
D) CT
Ans: c multiple sclerosis..if >50 years we think of temporal arteritis
Emad Askary 1st question: C. Assuming that the pain is around the eye or retrobulbar, it can be optic neuritis
(JM p.827). 2nd question

37. A male aged around 45 (I think. Somewhere between 40-50)


has been operated for fracture femur. He is heavy alcoholic and smoker.
24 hours post operative he in agitated and disturbed. Which of the
following it the next step in management?
A) Blood alcohol
B) Chest X-ray
C) Serum electrolytes, urea and Creatinine
D) Pulseoxymetrie
Ans:D

hypoxia

Emad Askary D. AMC handbook: "Acute delirium early after surgery may be due to a number of causes of which
hypoxia should be checked first by arterial blood gas analysis and by peripheral oxygen saturation findings."

38. A young woman presents to you after her delivery. She is


seeking advice for contraception. She want to breastfeed her baby for the
next 12 months and then want to have her second child. What is the most
appropriate management?
A) IUCD
B) Depo-Provera
C) Low dose progesterone
D) COC
E) Nothing to be done
Ans: C

39. Young girl has been prescribed sertraline has presented to you
regarding the use ecstasy. What will be your advice?
A) Recreational use of ecstasy is fine when taken with sertraline
B) Their use is contraindicated
C) Ecstasy augments the effects of sertraline
D) There is no contraindication.
Ans: B

it is not clear for me I need mor work up on it

Shilpi Singh So far, studies have shown that combining MDMA with SSRIs, including Zoloft, may not cause a
dangerous drug interaction. However, the combination still has consequences: taking the two together reduces
the effects of both the MDMA and the SSRI. Because Zoloft won't work as well if you're also taking ecstasy, your
depression may not be treated as successfully.
Saba Ismail b.. ecstasy(MDMA) causes increased release and decreased uptake of serotonin.its overdose
causes serotonin syndrome.its use with ssri may not cause a dangerous drug interaction but it does not augment
the effect of sertraline too. instead its administration with sertraline decreases the effect of both sertraline and
ecstasy. ecstasy also worsens depression.

40. Which of the following is the common site for endometriosis?


A) Round ligament
B) Ovaries
C) Uterus
D)Fallopian tube
Ans: B
Alain Diago Ovary60% uterosacral ligament 60%pouch of douglas 28% bladder 15% rectum 12% sigmoid colon
7% round ligament 5% and appendix 2%

41. 7 days after surgery for gangrenous appendix, lady developed


fever with rigors. What is the most likely cause?

A) Pelvic abscess
B) UTI
C) Wound infection
D) Chest infection
Ans: Abut may be C according to Kaplan page 25
Emad Askary Van Diesel, A. AMC handbook: Incidence of intra-abdominal abcess formation after
appendectomy for perforated appendicitis may be as high as 15%... Classical symptoms include spiking ever and
persisting mucous diarrhoea
Aarav Kumar A.....gangrenous appen.....wond infect will have singhs of local redness etc...fever n rigor shows
more generalised n toxic svenario.
Namrah Shafiq Abscesses may form within 1 wk of perforation or significant peritonitis, whereas postoperative
abscesses may not occur until 2 to 3 wk after operation and, rarely, not for several months. Although
manifestations vary, most abscesses cause fever and abdominal discomfort ranging from minimal to severe
(usually near the abscess). Paralytic ileus, either generalized or localized, may develop. Nausea, anorexia, and
weight loss are common.its c ? Merckx manual
Syead Murad Hossain C here. Pelvic abscess start producing fever around POD 10-15 where as wound
infection on POD 7 (ref Kaplan page 25, 2013 edition)

42. Man comes to you with pain and redness in his right scrotum
which has been gradually increasing over the last 5 days. Pic was given.
Which of the following organism is the most common causative organism?
A) Enterococcus
B) E Coli
C) Staph
D) Klebsiella
Ans:B
Fari Shaikh B if older
Fari Shaikh otherwise chlamydia

43. 3-4 hours after instrumental delivery, lady develops tachycardia. Her BP has
also decreased to 90/56 (I remember the BP in this question). On palpation
abdomen is not rigid or tender. The uterus is contracted and deviated to the left.
Vaginal blood loss had been minimal about 400-500 ml. The uterus is found to be
deviated to the left. Which of the following is the most likely cause
A) Uterine atony
B) Uterine rupture
C) Broad ligament hematoma

D) Uterine inversion
Ans: C
Minahil Obaid C for me- uterus deviated to a side,concoeled hemorrgahe,instrumental delivery
Noori Firdaus uterus is contracting so this rule out A &B could be C or D but in inversion bleeding cant occur
after 4hours usually occur in 3rd stage labour
Sunita Dembra Aryan Saif Mujtaba.....in usmle lecture yes uterus shifted in ruptured but here uterus is
contracted ,,,in rupture uterus fail to contracte

44. Young girl is brought to you. She has likes to play by herself, is
silent and doesn't socialize much. On examination she the only significant
finding is a patch of hair loss in the occipital region. There is hair of
varying length. Which of the falling is the most likely cause?
A) Alopecia
B) Impulse control disorder
C)Anxiety disorder
Ans: B

45. Woman who is diabetic and well controlled on Insulin and oral
hypoglycaemic comes to you with an ulcer on the dorsum of her right fifth
toe since 5 days. There yellowish purulent discharge from the ulcer. The
lateral aspect of her leg till her ankle is inflamed, swollen and red. What
will you do next?
A) IV Ticarcillin
B) Analgesics
C) MRI
D) Debridement of ulcer
Ans: C

then d then a

Shan Saleheen Mahwish Sheikh dear, from the above reference you will find stepwise approach, MRI is needed
for ulcer grading, as it could be Charcot's neuropathy (CNA), and, "As it is often difficult to establish the diagnosis
of osteomyelitis with microbiological sampling of affected bone, MRI has emerged as the investigative modality of
choice to distinguish osteomyelitis from acute CNA. C here first. (suspected Charcots neuroarthropathy e.g.
unilateral, red, hot, swollen, possibly aching foot)

46. Scenario of severe asthma attack. Asking which of the following


is the most reliable bedside test.
A) Use of accessory muscles
B) Spirometry
C) Number of words that can be spoken
D) Breath sounds
Ans: A
Adam Jp I think it is A bc b is not for acute or exacebation of asthma ( cant do maximal inspiration) , number words neither bc may have speaking problem or even cant speak bc of age, best in bed side would be A . m I right

47. Young female comes to you with complains of circumoral


tingling sensation following a minor surgical procedure which was done
under local anesthesia. Which of the following is the most likely cause?
A) Effect of local anesthetic
B) Hypocalcaemia
C) Nerve damage
Ans: A

48. Lady who underwent surgery in lithotomy position. Is unable to


dorsiflex her great toe . Which of the following is the most likely affected
muscle?
A) Extensor halluces brevis
B) Flexor halluces brevis
C) Tibialis
D) Soleus
Ans: A
Jibi Lee Daniel A.due to common peroneal nerve injury

49. Man from Somalia comes to you with soiling. On examination


you notice a fistulous tract. Which of the following is the most likely
cause?
A) Crohns
B) Ulcerative Colitis
C) Schistosomiasis
D) Perianal Abscess
Ans:D

50. Pic of smiling man standing upright. He is obese (Not morbidly).


He notices a swelling in his neck which has been increasing over the last 3
weeks or 5 months (Cant remember very well). On examination he has
redness on his face, neck upper torso and swelling both the upper arms.
Which of the following will you do first?
A) X-ray
B) CT Neck
C) CT Chest
D) USG

Ans:C
Emad Askary C. Oxford Handbook of Clinical Medicine 9th ed. p.526: Investigation for superior vena cava
obstruction: urgent CT. @Dr-Sarmad Sameer @Amir Fthi @Eman Khamis @Saifun Nahar @Dr-p Bn
Moslem Shahrdami Emad Askary
Medscape:
Plain radiography is often helpful and reveals a mediastinal mass in most patients.
Chest radiography may reveal a widened mediastinum or a mass in the right side of the chest.
A CT scan of the chest is the initial test of choice to determine whether an obstruction is due to external
compression or due to thrombosis. The additional information is necessary because the involvement of these
structures requires prompt action for relief of pressure.

51. A middle aged man comes to you with difficulty in breathing. He


also complains of chest pain. All his examination is normal except for his
saturation which his 85% and eGFR which is low. Which of the following
will aid you in diagnosis?
A) X-ray
B) USG
C)CTPA
D) V/q Scan
Ans: D but if renal function normal then CTPA
Ahmed Nagaty Saba Ismail Manu Prasad
CTPA is the best to diagnose PE except in pregnancy and renal failure( contrast)
Here the GFR is low so, it is C vq scan is the answer

52. Mother comes to you with 12 month old child. She has been
living with her new boyfriend. You notice bruises on her hand and arms.
On further examination you also see that she has bruises on her chest.
What will be the most appropriate next step?
A) Suggest to her that she is a victim of abuse
B) Ask her to meet with a social worker
C) Inform Child Protection Services
D) Inform police
E) Talk to her boyfriend
Ans: C

53. A lady who is 65 years of age comes to you for vaccination. Her
daughter is pregnant and she would be living with her now. Other than
her regular vaccination, which of the following will you advice her now?
A) Influenza
B) DTP
C) Varicella
D) TT
Ans: b
Noni RJ I couldn't find refference for Varicella Aryan Saif Mujtaba. If you could provide it, much appreciate. I
found on Immunization book 2013. Vaccination with dTpa is recommended for any adult who wishes to reduce
the likelihood of becoming ill with pertussis. Vaccination is particularly important if the adult meets the criteria of a
special risk group (see Persons in contact with infants and others at increased risk from pertussis below).

54. An elderly man comes to you with complains of increasing pain


in his leg. He was able to walk up to 200 meters but now is developing
pain in his right leg and the distance he covers has shortened. Now he
can walk only short distances. He has history of smoking and consuming
alcohol occasionally. His ABI was done which showed .8 and .2 ( Cant
remember which leg what). What is the next step in management?
A) Doppler USG
B) Walking exercise
C) Compression
D) Angiography
Ans: A

55. Pregnant lady comes to you with pain in the right upper
quadrant. USG shows gallstones. Which of the following is the most
important reason because of which you would suggest her
cholecystectomy?
A) At risk of carcinoma of gallbladder
B) At risk of developing gall stone ileus in subsequent pregnancy
C) Can have recurrent cholangitis
D) Can have recurrent gallstones

Ans: mostly c.may be d

Aswathi Hari Prognosis- cholecystitis in pregnancy


Complications can occur, including empyema, perforation, pancreatitis, and failure to respond to medical
management. Patients diagnosed with symptomatic cholelithiasis during the first trimester have a recurrence rate
of 92%; during the second trimester, the recurrence rate is 64%, and during the third trimester, the rate is 44%.
Compared with patients who undergo cholecystectomy, patients in whom surgery is delayed experience
increases in hospitalization, spontaneous abortion, preterm labor, and preterm delivery. Fetal loss occurs in 1060% of pregnant patients with gallstone pancreatitis.- medscape

56. Patient comes to you who has been visiting you and in the last
three appointments he has not paid his bills. What will you do now?
A) Ask receptionist to talk to him
B) Ask him to pay bills first and then seen him
C) See him between appointments
D) Refer him to hospital
Ans: mostley A ..May be D

57. Lady in dialysis ward. She is good to the nurses in the dialysis
unit and compliments them for their work. When she comes to the ward,
she is very rude to the ward nurses and abuses them. What is the
diagnosis?
A) Narcissism
B) Splitting
C) Electrolyte imbalance
Ans:B
The following question I don't remember very well
58. There was question about Romberg sign and something to do
with proprioception.
59. IgA nephropathy.
60. Some stem about vestibular system.
61. Carcinoma prostate
62. Breast carcinoma biopsy
63. Pericarditis

64. Handbook question 3.047 on restless leg syndrome. The stem


was very similar to the HB one but was asking for treatment.
This is the best I could come up with. Hope this helps. God Bless
You.

1. I have reconstructed the questions to the best of what I can


remember.
2. The stems questions I have recalled are around 40% or at best
50% of the question size which I got in the exam. There is a lot more
information in the exam questions.
3. Read the question very carefully. (I always failed at this)
4. A useful way I found to answer the questions is to read the first
part and then the last part of the question first. Then read the body of the
stem. Reading the first part of the question will give you a basic setting of
the question and reading the last part will you the direction in which to
think. After this read the body of the stem. I think this way, all that we
think in processing the question will be limited and wont think in any
unnecessary direction.
5. When preparing recalls, kindly spend some time and make good
ones. It really helps people and takes away a lot of confusion. It would
not be perfect but still. One line recalls would mostly be the assessment
of the person giving the exam and would not be very accurate. Kindly try
your best.
6. All the best and God Bless You. Would like to thank all the people
who have helped me on Goldstars.
Regards

MY RECALLS HANI,18TH
OCT,2014.

1. A 55 year old builder came with weakness of his left hand and leg not lasting for 12
minutes, same complaints couple of weeks back. he is taking only Aspirin, no other
drug, what will you advice him other than usual Mx (October 16 recalls)
a)Do not do strenuous exercise
b) Do not drive for 6 months

c) Add warferin
d) Add more anti coagulation
e)dont go 2 work unless allowed by ur gp.

Ans: answer is not included


JM 1333&1334 if we add dipyridamole to asprin it will be more effective but it is not
an option here. Warfarin can be used alone as it is contraindicated to be used with
asprin but it is not an option as well.

2-A 55 year old lady presents with fever, RUQ Pain, her sclera slightly yellow, USG
shows multiple gall stones, BP is Low 80/55, what will be next Mx
a) MRI
b) ERCP decompression
c) iv fluids
d) Intravenous cholangiogram
Ans: C

2. 3-A nurse has a needle prick injury from a known hiv positive pt. What would be your
nest step of management after sending blood sample.
a)give immunoglobulin
b) commence antiretroviral
c) wait for serology result
d) give both anti retro + immunoglobulina

Ans: b

JM1353

3. A 40 year old lady came with RUQ pain which resolves gradually. She doest have
any other symptoms. On CT scan following was found. What is the Dx

a) simple cyst
b) Hydatid cyst
c) Hepatoma
d) HCC
Ans:A
4. 4-A 61 year old man came with chest pain agitation and sweating, after doing ECG
following was found. He was given Aspirin, Morphine and Nitroglycerin. What will
you do next? (ECG of st elevation)
a) Coronary Angiogram
b) tPA
c) CKMB
d) IV Heparin
e) SC LMWH
Ans:If pt in tertiary then a,if in rural then b

5. 6-A 13 year old girl living outside for 6 months came with an unwanted sexual
relationship last night in a party.she doesnt want to inform anyone. What will be your
next step?
a) Call the police
b) Inform parents

c) Sexual assault service

Ans:C

6. A 6 yr child wid complaint of blood in urine now. He had 1st episode few months
back.bp 100/85.no edema. Urine reports show rbc +,protein +.
a) IgA
b) Nephrotic
c) Psgn

Ans: A
7. A girl wid persistent proteinuria on different occasions wid all labs normal.next?
a)renal biopsy
b) creatinine n cr ratio
c)usg

Ans:A
But if there is a choice of estimate the amount of protein in urine it is
before renal biopsy
Sepehr Sadeghi If two or more urinalyses show protein in the urine, the next step is to determine how
much protein is in the urine. This can be measured from:
A single urine sample collected at any time (a common and convenient method).
Urine that has been collected over 24 hours (a more exact but somewhat inconvenient method). (See
"Patient information: Collection of a 24-hour urine specimen (Beyond the Basics)".)

8. A pregnant lady came with severe frontal headache 15 week. She had bp 85/60.pulse
115/min. What would be your next step of Inv?
a) LP
b) MRI
c) FBE
d) Blood Culture
e) Usg abdmen

Ans: E
Dr-p Bn Saifun Nahar the obvious thing is headache because of hypovolemic shock or pre-shock .it can be
abruptio placenta or least likely EP...so logically we can choose us then fbe!

9. A man came with a swelling of her leg for 4 months. H/O mosquito bite. What would
be your Dx?

a) Fibroma
b) Nodular BCC
c) Melanoma
d)Keratoacnathoma
Ans: A
10- pic of hypopyon after cataract srgey.
11-- Swelling at post triangle for 1 yr. no systemic symptom
a. lipoma
b.lymphnode

Ans: mostly it is lymph node

12-Mother brings 16 yr daughter for sore knee. In private consultation with daughter (mom not around) she asks for chlamydia
screen. urine sample is taken and sent for checking chlamydia.it comes back negative. mom doesnt know about this urine test.
Mom calls n asks for consultation results . what will u tell her?
a).tell her the results
b.ask her to ask her daughter
c. tell her u cant reveal results over phone
d. plz come for consultation with daughter again
e. tell her u will reveal the results only to her daughter
Ans:E

13- a truck driver of a company had an head injury in MVA. Few days later,owner of company calls you to
know that when driver can come to work,he says he took the permission from driver to ask his gp abt his
health.wt will be your response?
a)tell him his present condition
b)ask him to talk to his lawyer
c)1st ask the driver about permission
d)make a written report 4 owner.

Ans:c
14-woman at 39 weekof gestation,cervix 2 cm fully effaced,head1cm above the ischial spine. After 4hours her contractions are
irregular but similar findings.
When you reassess her after 1 hour later what will lead to cesarian section?
a) Cervix 2cm, other findings same as before
b) Cervix 5cm , fully effaced head at ischial spine
c) Cx 3 cm , fully effaced head 1cm above ischial spine
Ans:A
15-a child diagnosed with pertusis for 2 wks,ur advice?
a)antibiotic for 5 days n exclusion

16-30 yr male recently moved to new apartment,neighbours complainin of his aggressive behaviour towards them,sometimes
shouting at them. Man says he sometimes hears voices of ppl callin him.
a)schizophrenia
b)personality doisorder
others were totally irrelevant.
Ans: mostly A.according to JM 486

Kirlus Armia Saifun Nahar in jm 486 causes of violence:schizophrenic psychosis:old male with paranoid
psychosis and young male prone to act violently and impulsively due to hallucinatory commands so i think A

10. A 28 yr old lady got atypical antisphychotics and become obese and DM 2. Whay
screening is important for her at this moment
a) Lipid profile
b) BP
c) Kidney
d) Cardiac

Ans: A
Adam Jp clozapine, olanzapine and quetiapine causes higher triglyceride olanzapine can cause DM
and weight gain too
Mahwish Sheikh Antipsychotics causes metabolic syndrome raised lipid and glucose,thats why check
lipids

17- pt on selegiline 4 parkinsons,now started sertraline 4 depression. Developed abd pain n diarrea.
a)reduce sertraline
b)stop both
c)stop sertraline
d)reduce selegiline

Ans: C cease sertraline as it is contraindicated to be used with seligiline as this increase the risk
of serotonine syndrome
18) recall of dementia. Add donepezil was the answer
19-primigravida of irish background in 1st trimester.how u ll screen 4 GDM?

A) GCT 50 mg 26-28 wks


B) OGTT 75 mg 20 wk
C)

GCT 75 mg 26-28 wk

D) OGTT 75 mg 16-20 wks

Ans: c
According to http://www.racgp.org.au/afp/2013/august/gestational-diabetes-mellitus/

Recommendations for routine testing and diagnosis of GDM


All women not known to have DM or GDM should have a standard 75 g OGTT at 2428
weeks gestation. This renders the glucose challenge test redundant. The ADIPS and
IADPSG criteria for the diagnosis of GDM are shown in Table 2.

Table 2. ADIPS and IADPSG criteria for the diagnosis of GDM 8,9

Fasting
PG: (5.1 mmol/L)
1 hour PG: (10.0 mmol/L)
2 hour PG: (8.5 mmol/L)
A diagnosis of GDM is made on one or more of these values

20-A man presents to your clinic. His BMI is in the overweight range. He is a smoker. He occasionally uses cocaine and
cannabis. Which of the following is the biggest risk factor for CAD in this patient?
A. Obesity
B. Cannabis
C. Age
D. Cocaine
E)tobacco
Ans: E the patient is only overweight but if he is obese then A
21)SLE pic wid malar rash,how u ll confirm the diag?
A) ANA
B) Antismith
Ans: B ..ANA support the diagnosis but is not specific

22)AS scenario wid NAPROXEN


23)AS scenario of lumbar spine,resistant to naproxen,next?
a) influximab.
b)sulfasalazine
Ans: A
According to http://www.racgp.org.au/afp/2013/november/ankylosing-spondylitis/

Management
Symptom management
Education and exercise
As with any chronic condition, patient education and support is vital in the management
of AS. Patient disease and medication information, as well as access to support groups,
can be found on the Arthritis Australia website (www.arthritisaustralia.com.au). A tailored
exercise and stretching program is recommended for AS patients and there are several
useful online resources (eg. www.nass.co.uk/exercise) and mobile apps (eg. iAnkSpond).

NSAIDs
NSAIDs are first-line therapy for symptomatic AS patients. Recent studies suggest that
regular NSAID use in AS slows radiographic progression more than on-demand use. 8 An
individualised assessment of risk of long-term NSAID use should be made in consultation
with the rheumatologist before long-term daily NSAID use is recommended.

Tumour necrosis factor (TNF) inhibitors


TNF inhibitor therapy has strikingly improved the quality of life for the more than two
thirds of AS patients with an inadequate response to NSAIDs.9 These agents are listed on
the Pharmaceutical Benefits Scheme (PBS) for active AS not adequately responsive to
exercise and NSAIDs, but not for non-radiographic axial SpA. As such, X-ray changes of
sacroiliitis are required. Randomised controlled trials, however, suggest better clinical
responses in early non-radiographic disease.
For IBP, all TNF inhibitors have similar efficacy and the choice of agent is often
determined by extra-articular features and patient preference. Approximately 94% of AS
patients commenced on treatment with a TNF inhibitor at Austin Health in Melbourne
remain on long-term treatment (personal communication). To date, there is little
evidence to suggest slowing of spinal fusion with TNF blockers.

Other DMARDs
Traditional DMARDs such as methotrexate and sulfasalazine have little effect in spinal
disease, but can be useful for an associated peripheral arthritis. Other biologic agents
available for rheumatoid arthritis in Australia have not been effective in

24-meningococal scenario in a child,next?


a)blood culture
b)penicillin
Ans: B

25) pulmonary embolism scenario wid low gfr,inv?


Vq scan

26-15 year old girl brought by mother with primary amenorrhea. She says she looks like all girls in her class in appearance, just
no periods. What will you do next?

a) Ask when breast development began


b) Ask abt pubic n axillary hair
c)Ask her if she has recurrent abdominal pain

Ans : A

27-old recall of holdin stick in left sided hip osteoarthritis.


28-Police brought a man who was found to b homeless with 3 bagagges containing all his worldly things. He wanted shelter to
police. Was saying that a Jewish Mafia group of hackers targeted him to kill. He thought that this was just becoz he imported
a pornography CD once. He was agitated.What would b the next most appro mx?
a.parental antipsychotic
b.Full clinical assessment
d.Immediate admission in Psychi unit
(No option of collateral history)

Ans: B

29-Girl dressed nicely, but depressed and not talking/eating. what to ask
a)suicidal ideation
b) eating habits

c) worried about school performance

Ans: A

Farwa Husain suicidal ideation...first and foremost thing to ask in a depressed patient

30-Palpitation and lightheadness after running for 15 kms which ecg


a)Af
b)Atrial flutter
c)V fibr
d) vt

Ans: D but the best answer is SVT


This question needs more work out as vt is not convincing
31-4 yr child wid foillowing ecg

a)digoxin
b)iv adenosine
c)face immersion in water wid carotid massage.
Ans: C

32-interstate truck driver got insomnia with tremors,palpitations and wt loss for last 3 months.wts next invx
1)tsh
2)urinary drug screening
Ans : A

33-Truck driver having on and off abdominal pain and bloating for the last 3 months , sometimes he has a foul smelling diarrea ,
he had a colonoscope since 1 year and it was normal wts best nest step ?
a. Repeat colonoscopy
b. Stool culture
c. Abdominal ultrasound
d. Sigmoidoscopy
e. CBC
Ans:B
34,35-two scenerios of pap smear in LCIS wid different stems.
36) old recall of capsule endoscopy.
37-A pt smoker plus occupational asbestos exposure as well.. Has pleural plaques & hyperinflation or CXR. Now dysopnea.
What will u do?
A.CT
B.bronchoscopy
C. Sputum exam
D. Per cutaneous biopsy
Ana: A
JM 524
38-Engineer wrking in a factory complaining exertional cough non productive that a wakes him from sleep , cxr hyperinflaated,
smoker. wt to do
a)Ct,
b)bronchoscopy
c)percutaneous pleural biopsy
Ans: May be A

the stem might be inaccurate. Hyperinflation suggests obstructive lung disease such as COPD and the diagnosis
of this condition is through pulmonary function test (JM p.1252)

39-itp scenario wid peteche.


40-Renal failure with unilateral artery stenosis

Ace
41-Patient on dialysis presents with dyspnea 5 days after dialysis. What would you do initially?
a. Contact dialysis team
b frusemide
Ans: A

42-.

(for urgent dialysis)

pt well during dialysis but after that HTN most of the time what give?

a.anti htn
b.frucemide 100 mg

Ans:A
43-A 50 y.o. patient after knee replacement. On the 4th POD the develops chest pain and dyspnea.
ABG: pO2 90 (N 95-100 mm) p CO2 31 (N 35-45) HCO3 21 (N 22-28) pH 7.32 (N 7.35-7.45) What is the next step in
management?
a. Oxygen by Hudson mask.
b. CTPA
c. ECG
d.CXR
44-female postpartum 4 d came with fever she delivered full term baby she was having small lesion in the vagina didn't need
suture cause of the fever is:
A)infection of her unsutured lesion
b)breast engorgement
c)endometritis
d)UTI
Ans: A

JM 1045but if there is offensive discharge then C

This picture is helpful anyway

45-50

y/o man wants prostat cancer screening but doesnt have any risk factor or
symptom

a-Talk about pons n cons of prostate screening


b-Psa n dre

Ans: A
46-Indigenous mom with 4mo old baby, not making eye contact, doesnt want to hold the baby during vaccination, doesnt
make any conversation,leave soon,wt lll u do?
a)send nurse 4 home visit
b)Check her behaviour on next visit
c)call her family
Ans: my choice A but important number of doctors suggest B
The question may needs more work out
47-Old

pt with abdominal distention constipation n use lagxative for long time,dre


normal dx?
a)Sigmoid volvulus
b)fecal impaction
Ans: as long as Dre normal exclude B..so A is the correct answer. But there
may be other choices with the complete stem to choose from
48-elderly wid h/o apendicectomy,currently on antiparkinsons drugs,developed
constipation n distention.v bad xray of obstruction,cud nt differentiate if SBO or
LBO.
A)small bowl adhesions
b)pseudo obstruction
Ans: the patient is taking antiparkinson drugs do B is the wright answer

49) cecal ca symptom- lethargy


50-Pt a week ago had MI and now on aspirin and clopidogrel. he has fever and pain in right upper quadrant several times,
every time he take antibiotics and bowel rest. on usg in this attack thick walled gall bladder,pericholysstic fluid wt to do
a)Change antibiotics,
b) immediate cholecystectomy,
c) ercp and sphintrotomy,
d)percutaneous drainage
Ans: D

Hani Saleem For patients at high surgical risk, placement of a sonographically guided, percutaneous,
transhepatic cholecystostomy drainage tube coupled with the administration of antibiotics may provide definitive
therapy.D
51-female with history of mass lateral to pubic tubercle, presenting with nausea, vomiting, abd distention, invest.:
a-usg swelling
b-ct abdomen
c-xray abdomen
previously discussed in the same month recalls
52-Dvt from one wk on warfarine, developed perforated PU, wt next:
A-stop warfrine and operate after 2 days
b-stop warfarine, IVC filter, oprate
c-stop warfarin, start heparine , operate
d-stop warfarine, give FFP, operate
Previously discussed
Ans:D
53-incisional hernia.
Ask 2 Stand n cough.
54- 17 year girl started to have sexual relationship with her boyfriend recently and asking about chlamydia screening, ur
suggestion
a.chlamydia can be asymptomatic
b. her BF may have asymptomatic symptoms
Ans:A

Nahyan Kabir A as both of them can have asymptomatic infection..where as b is just referring to the BF
55-64 old lady come with sadness and tearfull 6 weeks when her husband died she had same episode when she wau 23 years
when she had still birth which of the following stop her symptoms promptly
a-citalopram
b-venlafaxine
c-ect
d-olanzapine
Ans: May be A or B..but I am not convinced as this is normal greif

Emad Askary Ireen Baker Ahmed, The stem is incomplete or some choices are missing. She has lost her
husband 6 weeks ago, so it can be normal bereavement. See JM p.34. The best measure for normal
bereavement is appropriate consultation.

56-another similar kind of scenario with venlafaxine as answer after posttrauma depression n insomnia
57-scenerio of fibroadenoma,1st inv? USGJM 956

58-A 40 yr old man with who is a smoker and a known alcoholic, comes for a general check up. His father recently had a stroke
8 months back at the age of 70. His BP is 145/90. His lab findings were given,cholesterol 7. Wht is most imp cardiovascular risk
in his case?
a-smoking
b-alcohol
c-HTN
Ans: Mostly Amay be c

Saifun Nahar Established facts 10,11,12


Major risk factors for CAD include:
increased LDL cholesterol + reduced HDL
cholesterol
ratio LDLC/HDLC >4
Risk increases with increasing cholesterol levels (90%
if >7.8 mmol/L)
TG levels >10 mmol/L increases risk of pancreatitis
Management should be correlated with risk factors
10% reduction of total cholesterol gives 20% reduction
in CAD after 3 years......jm 1285

59-Epigastric pain radiates to the back settle after 2 hours npt is fine what to
do?
a-Lipase
b-Gasteroscopy
c-ecg
d-ct
Ans: A serum lipase which is more specific than serum amylase for acute
pancreatitis
Kirlus Armia pain in acute pancreatitis:Tummy (abdominal) pain - just below the ribs is the usual main symptom.
It usually builds up
quickly (over a few hours) and may last for several days. The pain can
become severe and is typically felt spreading through to the back. The
pain may be sudden and intense, or it may begin as a mild pain that is
aggravated by eating and slowly grows worse. However, it is occasionally
possible to have acute pancreatitis without any pain. This is more
common if you have diabetes or have kidney problems.
Kirlus Armia pain in chronic pancreatitis:Tummy (abdominal) pain - just below the ribs is a common. The
pain is typically felt spreading
through to the back. It tends to be persistent and may be partly eased
by leaning forward. It may be mild at first but can become severe.
Eating often makes the pain worse. This may lead to your eating less and

then losing weight. The pain can be intermittent - so not present all
the time. Note: around 1 person in 5 with chronic pancreatitis does not have any abdominal pain

60-antiglomerular basement membrane scenario.


61- Orthostatic hypotension 1st line treatment.

63-pt is confused postoperative n want 2 change his will,n want u


2 witness the will,wt will be ur response.
a)ask registrar 2 witness
b)dont witness
c)witness in presence of his lawyer

Ans:B
64- CTG Of variable decelerations given,in primigravida near term, wid ruptured membranes n 5
cm dilatation,meconium stained liquor,next?
a)c sec
b)fetal scalp sample
c)continue ctg monitoring
Ans: B
According to JM 1036
65- PMR askin abt inv? ESR
66- A man wid abdominal pain,having fever 39,pain then became pelvic,on rectal
examination,there was little tenderness.next?
a)laproscopic drainage n appendicectomy
b)open drainage
c)ct
Ans: C according to HOMCQ 2.096q 162pg

But a large number of doctors suggested A..but I am not conconvinced


67-A 3year old child is brought by his mother as he is complaining of pain in RUQ fro the
last 2 years, recently she has noticed mass in RUQ , What is Dx?
a) Neuroblastoma
b) hepetoblastoma
c)wilms tumor
d) HCC
Ans: C

See Kaplan P 220 &221


HOMCQ 2.142 p.197

Moslem Shahrdami Medscape:


Wilms tumor, or nephroblastoma, is the most common childhood abdominal malignancy.
The most common manifestation of Wilms tumor is an asymptomatic abdominal mass; an abdominal mass
occurs in 80% of children at presentation. Abdominal pain or hematuria occurs in 25%
68-pt with ra , on CBC, all 3 cell lines r decreased.cause?
a)aspirin
b)diclofenac
c)hydroxychloroquine

Ans: Both B & C cause aplastic anaemia..but it was agreed to choose B.I do n ot know the cause

69-Gyanecomastia pic lateral view


teenager plays footballactive in sports. He has hx of asthma and on fluticasone. What is
the cause of gynaecomastia
a) Anabolic/ steroids use
b) fluticasone inhaler
c) Idiopathic

Ans: Cl

But if the question says that he uses anabolic steroids then A


Below pic from HOMCQ
See also JM1050

70- A lady is set to sit the AMC on the 12th of May but turns up to your office 2 days after
asking for certification of her retrospective illness to explain to the AMC why she could not
sit. She is currently asymptomatic and becomes agitated when you refuse to comply with her
request. She starts threatening to report you to the board. You in the meanwhile tune her out
in your mind and play the radio. Her behavior is a result of:
A. Malingering
B. Narcissistic Personality Disorder
C. Borderline Personality
D. Somatatisation of the Rectum

Ans: C

71- Boy with photophobia, neck stiffness. Csf finding given , lymphocyte high, slightly increased neutrophil, protein high,
glucose normal. What will you do?
a) iv ceftriaxone
b) iv acyclovir
c) do mri
d) supportive
Ans: D viral meningitis but if neutrophils are more than 100 suspect bacterial and then Aif csf contains blood suspect
SAH and do CT
JM 270-272

72-.Female with cycle 4-6 now amenorrheas 6 wk with + ve urine preg now vag bleeding Us done and found 12 mm
endometrial thickening,empty uterus, no fluid in doglus of pouch, adenexa free, corpus luteal cyst in the ovarydx:
a)ectopic
b)false positive pregnancy
c)complete abortion
D. Incomplete Abortion

Ans: I selected C

Nabiha Binte Ali C here.


In EP 98.6% cases abdominal pain is present. Here pain hasn't been mentioned & adnexa free, no fluid in
Douglas. So it can't be ectopic pregnancy.
According to medscape if a married female presents with amenorrhea & vaginal bleeding then think about
pregnancy related issues. So false preg is not possible.
In incomplete preg retained products would be present in uterus.
Emad Askary JM p.970: Trans-vaginal ultrasound can diagnose EP at 5-6 weeks (empty uterus, tubal sac, fluid
in cul-de-sac). So it is not EP. The history of amenorrhoea, vaginal bleeding and the mentioned ultrasound, all
support complete abortion.
73-a 65 year old lady with bloddy discharge...no lump...sister has ca breast..
a-Benign duct papiloma
b-intraductal carcinoma
Ans: I am with A
See HOMCQ 2.118 page 85 JM 956

Emad Askary A. JM p.955: "The majority of patients present with breast lump." Although it is apparently benign,
JM on p.956 says this condition must be differentiated from infiltrating carcinoma through the excision of the
affected breast segment.
74-50 years old previously healthy man .comes with palpitations. Irregularly irregular .what will u find on cardiac exam
A-normal cardiac exam
B-S4
C-MR findings
D-PROMINENT a on jvp
Ans: A..lone or isolated atral fibrillation
JM761
75- patient with complaint of confusion . He was takingindapamide and ACEI. His labs showed serum.Na 120 mmol/l. How will
you manage?
a. Restrict oral intake to 500ml/day
b. Give hypertonic saline
c. Stop indapamide
d. Stop ACEI
Ans: B..then stop indapamide as one of its side effects is hyponatremia but first we have to treat confusion

76-Old recall of anaphylactic skin reaction in child.


Oral promethazine
77-Old man, back pain one week ago while working in the garden, now : point tenderness, low grade fever, what is the most
likely diagnosis?
a. Discitis.
b. Herniation.
c. Sepsis.
d.Vertebral fracture
Ans:A

Amir Maibody A note that disc herniation is rare in elderly due to rigidity of anolous a low grade fever will not
lead to inf in this scenario but more to an inflammation
Shilpi Singh Working in a garden cant lead to discitis... There is debate as to the cause, although hematogenous
seeding of the offending organism is favored as well as direct spread. It is important to differentiate between
spontaneous discitis which is usually from hematologic spread from a urinary or respiratory infection versus that
from a post-operative complication which usually involves skin flora such as staph aureus.
78-male come with cervical injury ,hypotension ,HR=50,so2 =90% in room air
what to do
a-iv colloid fluid
b-oxygen
c-trendlenburg position
d-adrenaline
e-atropine
Ans: A..if asked what is next then oxygen
79-scenerio of OCD,asking management?
Exposure n response control
80- 68 yo comes for a routine visit and says her daughter is pregnant. What vaccination would you recommend in addition to
influenza vaccine?
a. DTPa
b. Hemophilusinfluenzae vaccine
c. Pneumococcal vaccine
Ans:A
81-recall of deep injury in farmer wid toxoid dTPA 3 mon aback 4 a similar injury,no past h/o immunization,next?
DTPA vaccine + Ig
82-milestone recall
83-pic of divarication of recti in a man,wid positive shoulder raising.asymptomatic.mx?
a-physio
b-srgery
c-hernioraphy

Ans:A
84-HB scenario of ct head wid tumor in old man wid confusion,same pic,same stem,same options.
85-man wid pain n redness of rt leg,also has ulcer on left malleolus,cold limb n pulses weak on rt side.major cause of
condition?
a-redness
b-ulcer foot
c-buerger sign
d-pain
e-pulselesness
Ans: E
86-another scenario of decreased ABI on left side,<0.3 in smoker.dont remember exact scenario,wt next?
a-angiography
b-walking n exercise
c-stop smoking
Ans:E

Mahwish Sheikh here critical limb ischemia ABI <0.3 too low..patient needs angiography

87-man wid sciatic nerve injury,SLR 30 degree,what is lost?


a-knee reflex
b-ankle reflex
c-hip flexion
Ans: B
88-Man come with arm swelling after 1 day work with chainsaw. Dx?
a. subclavian vein thrombosis
b. lymphatic obstruction
c-cellulitis

Ans:A
http://www.ncbi.nlm.nih.gov/pmc/articles/PMC2967689/

Michael Gamal A, it's Paget-Schroetter Syndrome


89-21 year man present you with ankle sprained. Beside treating ankle problems, wt u can screen him
a .testicular ca
b.HTN
c.DM
D.lipid
ANS:B..JM 68
90-child moving in house confused in morning. cause
a-ketotic hypoglycemia
b-DKA
Ans: A

Based on wikipedia:
The typical patient with ketotic hypoglycemia is a young child between the ages of 10 months and 4 years.
Episodes nearly always occur in the morning after an overnight fast, often one that is longer than usual.
Symptoms include those of neuroglycopenia, ketosis, or both. The neuroglycopenic symptoms usually include
lethargy and malaise, but may include unresponsiveness or seizures. The principal symptoms of ketosis are
anorexia, abdominal discomfort, and nausea, sometimes progressing to vomiting.
91-woman is challenged with combined oestrogen & progesterone, but does not get withdrawal bleeding. Which organ is most
likely to be affected?
a) Overies
b) Uterus
c) Pituitary
d) Hypothalamus
e)Fallopian tubes

Ans: b (if the question asking about what organ is tested)

Mary Melad Guys sorry if quest ask main site action hypothalumus if progest test uterus

92-temporal lobe epilepsy old recall wid blank staring n hallucination of oceans wave.

93-old recall of insuficient breast feeding cause? Infrequent feeding


94-baby born wid cataract,micricephy n heart defect- rubella
95- pt on lithium,developed akathasia,seeking advice?
a-decrease dose of lithium
b-taking benztropine will improve akathasia immediately
c-propranolol will relieve it.
Ans:A JM 483
96-pt wid ecg of AF n some other finding,which i cud nt get. Tough one.inv?
a-echo
b-holter
c-cxr
Ans: A ..trying to find the cause.
97-Pregnant lady with multiple stores in GB why u recommend cholecystectomy ??
a) risk of gallstone associated complication later in preg
b) risk of cholestasis.
Ans: A

Emad Askary Shona Amber, I think the stem is inaccurate, since Oxford Handbook of Clinical Specialties 8th
edition on page 38 says "surgery should be reserved for complicated non-resolving biliary tract disease during
pregnancy, as in > 90% the acute process resolves with conservative management.
98-peptic Ulcer bleeding episode..two attempts done for hemostasis endoscopically but no success..next?
a-Operate n Suture
b-Again try endoscope
c-Gastrectomy
d-heater probe
Ans: D
99-pt wid chronic duodenal ulcer,now developed pyloric stenosis,wt ques u will ask?
a-vomiting immediately after meal
b-vomiting 1 hr after meal
Ans:B

Saifun Nahar vomiting may be intermittent and usually occurs within 1 hour of a meal.medescape

100- old recall of typical chronic fatigue synd,wt will u ask?


Wt u think is the cause of ur condition.

101-ecg

of sinus bradycardia given,in a lady wid central obesity,wt next u ll do?

a)echo
b)tsh
c)urine drug
d)atropine
Ans:b
Mybol Sous By exclusion: Pacemaker is recommended from 2nd degree HB , atropine is recommended in sinus
bradycardia < 40 b/minute. So, TFTit was an answer to another similar question

102-A patient came with palpitation, breathlessness. He has done ABG and shows PH: 7.54,
Po2: 102, Pco2: 28, what would you do next
a) Rebreathing mask
b) O2 high flow
c) Na bi Carb
d) Intubate
Ans:A
Shan Saleheen respiratory alkalosis due to hyperventilation

103-11-A 25 year old man came after MVA with horeseness of voice, loss of aortic knuckle,
and widening of mediastinum, BP stable.(Same recall)
a) Transcutenous Echo
b) Transthoracic Echo
c) CT angiogram
d) MRI
Ans:C
Noni RJ C. If the patient stabilizes in the ED and does not require emergent operative treatment, a
chest CT with contrast is performed to define the extent of thoracic injury and exclude aortic rupture.
If the patient is unable to undergo CT, due to the need for immediate operation, transesophageal

echocardiography can be performed in the ED or operating room to assess the aorta and
heart. Adam Jp
Sepehr Sadeghi There has been a shift from an invasive (x-ray aortography) to a noninvasive
diagnostic strategy for evaluating suspected thoracic aortic dissections [23]. In the 2000 IRAD
review, most patients had multiple imaging studies performed (mean of 1.83 per patient); the initial
study was computed tomography (CT) in 61 percent, transesophageal echocardiography (TEE) in
33 percent, aortography in only 4 percent, and magnetic resonance imaging (MRI) in only 2 percent
[7]. Recent experience suggests that CT is even more prevalent as the initial study of choice,
especially due to its widespread availability in the emergency department setting.

104- A 4 yr child wid fever 39, cough n breathlessness,nasal flaring.


a)iv bezylpenicillin
b)flucloxacillin
c)adrenaline
Ans: A pneumonia
105-pt on sertraline,developed diarrhea n abd pain,next?
a-switch 2 venlafa
b-switch 2 another ssri
c-switch to tca
Ans: B

Rumaisa Shaukat B hb qs similar


Rest of the ques were almost new in a sense that they were old recalls wid new n twisting options.but if you
have gud approach n understanding of each recall of past 6 months,you can solve that twist. if you have prepared topx of
recalls v well then time management is nt that big issue during exam as told by every1 on goldstar, v gud luck 2 all. Plz
remember me in prayers. Special thanx to Rumaisa dear,she is a blessing 4 goldstar.stay blessed dear. N yes to saifun
naihar,my last month recall partner: )

MY RECALLS 13/10/14
I have tried to recall as much as I can I hope this recall will be helpful to all the
goldstars. Please read the recall topics well coz it helps especially the last 3
months before your examdont forget about the handbook too ;) In the real
exam questions are really long.some are good coz more info is giventime
management is important tooall the best goldstars!! Please keep me in your
prayers if possible.;)

1. Uterine prolapse caused by what ligament?


a) Round ligament
b) Uterosacra ligament
c) Broad ligament
Ans: B
2. Salmonella outbreak in your GP setting. What will you do?
Weird options but I choose to compare the food that the people with
disease ate and the ones not infected.kinda case control type
3. Baby with cleft palate . mother took carbamazepine during pregnancy and
drank wine ( standard drink) daily. What is the cause of cleft palate in
baby?
a) Alcohol
b) Genetics
c) Carbamazepine

pic something like this but


obvious cleft palate.
Ans: B
4. Hb ques cerebral tumour page 222 MCQ 3.042 exactly the same!
5. Old lady with right lower lobe pneumonia ( cxr given with right lower lobe
haziness) . symptoms she had was productive cough with sputum , blood
tinged. SP02 was 92 % . otherwise she was well. Treatment with choices
of abx
a)
b)
c)
d)

Penicillin
Amoxycilin / clavulanate
Doxycycline
Azithromycin

CXR was something like the one in the first one NO lateral view was given
Ans: If >65 years then A
If >65 and can tolerate oral or <65 then B

6. Woman at 39weeks of gestation in labour. Her cervix is 2cm and fully


effaced , head is at 1cm above the ischial spine. After 4hours her
contractions are irregular but similar findings.
When you reassess her after 1 hour later what will lead to cesarian
section?
a) Cervix 2cm, other findings same as before
b) Cervix 5cm , fully effaced head at ischial spine
c) Cx 3 cm , fully effaced head 1cm above ischial spine
Ans:A

7. Slapped cheek pic.. a very huge pic of a girl smiling widely with her
cheeks pinkish
Scenario was mother came to you with her daughter as in the pic asking
when she can go to school
a) Immediately
b) After 2 days
c) Some other weird options

pic was something like this.


Ans: A
8. Gyanecomastia pic lateral view
The scenario was about teenager plays footballactive in sports. He has hx of
asthma and on fluticasone. What is the cause of gynaecomastia
a) Anabolic/ steroids use
b) Asthma
c) Idiopathic

pic was something like this.


Ans: C(previously solved at the recalls of this month)
9. Ankylosing spondylitis scenario with obvious bamboo spine xray. Patient
was on naproxen and physiotherapy for 2 years but pain is not improving.
What is the next treatment of choice?
a) Indomethacin
b) Sulfasalazine
c) Methrotrexate

the xray given was


something like this.

Ans: B

JM 385

10.STEMI scenario in metropolitan hospital. Patient had chest pain for 2hours.
Asking for next step of management.

Ecg something like this..


a) Tpa
b) Coronary pci
c) Thrombolysis
ANS: B
JM 418
Marien Ammy B.should be done within 90 min of arrival if patient presents with 1-3 hrs of
symptoms (if available ) if not available thrombolysis can be done

11.Scenario of young male athlete after running for several km he developed


palpitation and lightheadedness. There were 5 ecg strips given .
Answer options were like rhythm A,B,C,D,E
ECG strips were like theseI cant find the exact ones but it was confusing
psvt or svt strip!!
A)

B)

C)

D)

E)

Ans:

we must search for SVT

ECG

so it may be A

12.10 day old baby , thriving well with the pic given below. Asking for
diagnosis.

Something like this picture but the baby was really cute ;) I cant find the
exact one but that baby had yellowish discharge on the right eye ONLY. No
swelling of the eyelids and redness and eyes were closed
a)
b)
c)
d)

Nasolacrimal obstruction
Gonorrhoea
Chlamydia
Some other weird options
Ans: A
Rehenuma Tarannum Lazuly this picture is of nasolacrimal duct obstruction..sorry lost the
link...and chlamydia infection is kind of more severe than this
Nasrin Akter Maya nasolacrimal duct obstruction...usualy uniocular, no conjunctival or lid
congestion if not infected 2ndarily.
Chlamydia n gonorrhoea both eye usually n profuse purulant dischage with lid sewelling.

13.Temporal lobe epilepsy scenario.visual hallucination of waves coming


towards the patient and staring blankly at the ceiling. Asking for location
of lesion.
a) Occipital lobe
b) Medial temporal lobe
c) Frontal lobe
Ans:b
Noori Firdaus it is called as Macropsia seen in medial temporal lobe epilepsy

14.Capsule endoscopy in the answer options : 3 questions.


One was about this old lady having bloody diarrhea..colonoscopy done
but cant go further than hepatic flexureasking for next investigation?
a) Gallium scan
b) Capsule endoscopy
c) Thalium scan
d) Ct scan
Ans : B mostly
http://emedicine.medscape.com/article/197525-overview#a1

Shan Saleheen CT colonography (virtual colonoscopy) is a sensitive non-invasive technique for


diagnosis of tumours and polyps greater than 1 cm that can be used if colonoscopy is incomplete or
high-risk (Davidson 21st 911)

15.The other ques was about this male had endoscopy and colonoscopy done
but normal findings. Asking what is next ?
a) Capsule endoscopy
b) Gallium scan
c) Some other weird options
Ans:A
16.A scenario with all the blood profile given with the norms by the
side.from what I remember is Hb was low, Platelet normal, Reticulocyte
0.1 , ESR was raise
They mentioned there was roulex formation.
a)
b)
c)
d)

Multiple myeloma
CLL
AML
ALL
Ans:A

17.Thyroglossal cyst scenario with a pic similar picture as Hb page 68 MCQ


2.072 but no description given except lump is painless and not growing
bigger. Asking for next investigation
a) Usg
b) Ct neck
c) X ray
Ans: A .or may be b as lng as it is available according to
http://www.patient.co.uk/doctor/Thyroglossal-Cysts.htm
The diagnosis can usually be made from the history and a careful neck and physical
examination. Always palpate the thyroid gland during the physical examination. If the
gland cannot be palpated, ultrasonography, thyroid scanning or CT scanning may be
helpful. Diagnosis can usually be achieved on an outpatient basis.
Ultrasound is the most commonly used investigation.[9] Ultrasound and CT scanning are the
investigations of first choice:

Ultrasound can distinguish solid from cystic components.

CT scanning shows the capsular enhancement.

18.Dothiepin overdose scenario.this old lady was depressed or something


then swallowed a lot of dothiepin tabletsthen she told her husband about it
and he took her to ED. What is the most likely complication in this scenario?

a) Respiratory depression
b) Cardiac arrhythmia
c) Extra pyramidal symptoms
Ans: B
19.A lot of meningitis questions so please read properly.
One scenario of viral meningitis CSF given with lymphocytes.glucose and
protein normal.asking for management
a) Antiviral
b) Symptomatic and observe
c) Some weird options
Ans: B
20.Mother came with neck stiffness and photophobia. Son had URTI. CSF
picture was viral. Asking for causative organism
a) Adenovirus
b) Enterovirus
c) RSV
d) Streptococcus pneumonia
Ans: B JM 270
21.Aboriginal woman came brought her child for immunisation but she didnt
make any eye contact. After immunisation she took her child and left
immediately. Asking what is next?
NO CULTURAL OR SOCIAL WORKER in the option.
a) Send nurse for home visit
b) Check her behaviour next visit
c) Ask her to bring family members for next visit
Ans: A
22.Neuroleptic malignant syndrome scenario.
a) Diazepam
b) Dantrolene
c) Some other drugs.sorry it was a long scenario I cant remember
everything in detail.
Ans: B
JM 483
23.Acute dystonia scenario.a guy was brought in by the friends with
oculogyric crisis ( eye rolled up) and the patient was embarrassed. Asking
what is the treatment?
a) Diazepam
b) Benztropine
c) Some other drugs.
Ans: B.JM483
24.Bronchiolitis scenario..9months old baby was brought in by mom
symptoms mentioned and asked for diagnosis.quite straight forward.
25.A lot psychiatric questions.about antipsychotic drugs.the reactions
sorry I really cant remember coz it was very long scenarios.plz study
psychiatry properly especially the drugs!!

26.Nifedipine and SSRI interaction scenario.patient was on


fluoxetine.asking for what caused the symptoms mentioned in the
scenario..
Weird options.a) fluoxetine
b) drug reaction
Ans: B.Fluoxetine increases nifedipine conc so nifidepine dose should be reduced...
http://www.holisticonline.com/remedies/depression/dep_interactions_SSRI.
htm
27.student from china had rumbling stomach..no other symptoms
mentionedseems like IBS
scenario.he is having exam in few days
time.Asking for treatment?
a) Antidepressants
b) Reduce fat diet
c) Increase water intake
Ans: almost all the members answered Abut I am not conviced
28.Children milestones..2 questions
One is 12monts old child..what indicated developmental delay?
Cant understand NO was in the option ;)
29.4 years old child.what indicates developmental delay?
a) Cant ride two wheel bike
b) Cant draw a face
c) Cant dress without supervision
Ans: A
JM 845
30.Borderline personality disorder scenario..this woman with broken
marriage.at the office always shouts at her collegue.
31.23 years old female with criminal offences and was in prisondrugs abuse
....stays away from family.asking what is the diagnosis.
a) Borderline personality disorder
b) Conduct disorder
c) Substance abuse
Ans: cif antisocial available it may be the best choice
32.Fluid resuscitation ques.scenario of this old lady was found lying on the
ground by her neighbours.she was drowsy and dehydratednoticed
dark urine when urinary catheter inserted.nothing mentioned abt her
being diabetic.weight given : 42kg
Blood investigations given : Na normal , K normal , creat high, urea was
not mentioned , blood sugar level was not mentioned.
What is the next step of management?

a)
b)
c)
d)

Normal saline with urinary output 2ml/kg/hr


Dextrose 1/5 saline with urinary output 1ml/kg/hr
5% dextrose with urinary output 2ml/kg/hr
Normal saline with urinary output 1ml/kg/hr
Ans: D
Emad Askary Adam Jp, D, is the most appropriate choice, but not the most perfect one. 0.9% saline is
more appropriate than 5% glucose as for fluid resuscitation, as it will remain predominantly in the blood.
Moreover, since we do not have blood glucose level, it is better not to administer any glucose. Aim
urinary output > 0.5 mL/kg/h during management of hypovolaemic shock.. Elevated creatinine can be
due to pre-real acute kidney failure that must be treated with 250-500 mL saline over 30 min. (Oxford
Handbook of Medical Sciences 9th ed. pp.680 & 804 & 848)

33.Incisional hernia scenario..this guy had midline incision and developed a


swelling on the incision lineno picture given.asking what will help you
diagnose it ?
a) Ask the patient to stand and cough
b) Ask the patient to lie down and cough
c) Ask the patient to lie down and raise the head to chest
Ans: B
Noor Eldeen INCISIONAL HERNIAS EXAMINATION. Lay the
patient down, and put your hand through the
weakened area in his abdominal wall to feel the
size and shape of his hernia. It may be elliptical,
or irregular, and he may have more than one. Ask
him to raise his head and shoulders off the couch without using his arms. This will fill the sac and
show you its true size.

34.CRAO scenario with cherry spot picture given.


The old lady had pain in eye for 3 hours.came to ED now .what is the
management?
a)
b)
c)
d)

IV acetazolamide
Globe massage
Timolol
Pilocarpine
Ans: A..but globe massage is one of the steps .JM 825

The picture given in exam was something like this..


35.Post streptococcal glomerulonephritis scenario given..with a boy having
urti and noted to have ankle edema. Bp was a lil high for his age.
Urine dipstick shows proteinuria and hematuria.
What is the diagnosis?
a) Ig A nephropathy
b) Post streptococcal glomerulonephritis
c) Thin basement membrane disease
Ans: B
36.Good pasture syndrome scenario.with this guy having bibasal crackles ,
proteinuria,hematuria..asking what is the diagnosis?
a)
b)
c)
d)

Glomerulonephritis
Thin basement membrane disease
Anti glomerular basement antibody disease
Ig A nephropathy
Ans: C

Anam Tahir good pasture syndrome also known as anti glomerular basemtn antibody syndrome...C

37.Sciatica , lumbar radiculopathy scenarious..so read all these well

38.Guy with back pain after lifting heavy things. Pain resolved after a week.
Now develops back pain and tenderness at L2 region. Noted to have fever
37.8 degrees. What is next step of management ?
a)
b)
c)
d)

Ct scan
X ray spine
MRI spine
Bone scan

Ans: mostly b.As

Plain X-rays of the lumbar spine are not routinely


recommended in acute non-specific low back pain (pain
<6 weeks) in the absence of 'red flags' as they are of
limited diagnostic value and no benefits in physical
function are observed.' J.M P.381

We have to review JM for investigations 381 and 376 for red


flags
39.Old lady with neck of femur fracture. She is on clopidogrel. Her
coagulation profile is normal. What is the next step?
a) Stop clopidogrel immediately and surgery after one week
b) Transfuse ffp and surgery now
c) Some other options I cant remember
Ans: If surgery is urgent and the patient has a stent then continue
clopidogrel and give platlets.
If surgery is not urgent and no stent then A..
HOMCQ 3.144 P.268

40.Couple came for advice. Male sperm count 19 million, 40 % motility , 65%
abnormal forms.asking what advice will you give to the couple?
Options were totally different and weird from the usual recalls
a) Spontaneous pregnancy chance is not reduced
b) Spontaneous pregnancy is unlikely
c) They will definitely get a boy as a child
d) Some weird options
Ans:A.JM 1082
41.Child wandering around the house early in the morning noticed by
parents.urine dipstick ketones 3+ asking for diagnosis?
a) DKA
b) Cant remember other options.sorry.
Ans: hypoglycimic ketonuria..previously answered in this month
42.25years old female with lump on her breast..fibroadenoma
scenario.asking for what is the investigation?
a)
b)
c)
d)

Mammography
Usg
Fnac
Ct scan
Ans: B after that Fnac.JM 956

43.45 years old lady with breast lump.firm , round lump, painful.asking
for diagnosis?
a) Breast carcinoma
b) Breast cyst
c) Fibroadenoma
d) Sebaceous cyst
Ans: B.JM 955

44.Dementia scenario..seems like lewy body dementia but not with the
usual symptoms..asking what will aid into diagnosis?
Answers were really weird optionsjust brush up on all types of
dementia
45.Uterine rupture scenario..symptoms were clear and obvious with
tenderness over uterus, increase fundal height, rigidity of uterus..
46.Pregnant woman in labour room just deliveredblood loss 1000ml, not
that hypotensive, tachycardic, normal placenta removal. Im
synthometrine given..baby was delivered and need to be
resuscitated..it was a long scenario.cant remember other
details.asking for what is the diagnosis?
a) Uterine rupture
b) Uterine inversion
c) Uterine atony
Ans: C mostly..
47.52 years old woman with hx of tubal ligationno other operations done
before..complaints of hot flushes.no other symptoms mentioned. What
is the management?
a)
b)
c)
d)

Cyclical estrogen and progesterone


Combined estrogen and progesterone
Hormone replacement therapy
Estrogen alone
Ans: AJM 978.HOMCQ 3.217 P. 307
Emad AskaryShilpi Singh, A. The patient does not complain about
amenorrhoea, so she must be experiencing perimenopausal symptoms.
See JM5 p.987 figure 96.2

48.Young boy with asthma symptoms after exercise in school. Has history of
eczema. On auscultation u can hear expiratory wheeze..otherwise child
is well. Asking for diagnosis?
a) Asthma
b) Bronchitis
c) Urti
Ans: A

Thats all I can recall for now.am sorry if these ques are not in detail as the
exam ques were very longsome were informative.some were to distract.
I got a lot of ethical and statics theory based ques.no calculations..except for
one which had FEV1 value and FVC < 70 % in it and sort of asking
incidence.cant really remember it..

New ques were there but long scenarios.cant really remember it.
Ecgs were not the same typical ones lk in recalls .so go through different
websites to get more exposure.
All the best GOLDSTARS!! Please keep me in your prayers ;) I tried my best to
quickly recall as much as I can and post this recall before I go for a holiday.so
that all of you can continue studying without waiting for me ;)
Thank you to all the goldstars for helping me out throughout my preparation for
this exam. I hope that I will pass this exam and so all of u ;)

Recall 18 October 2014


1.
2.
3.
4.
5.

Hypopion pic
Keratitis scenario
Ct of ca pancreas.ask for inv..ercp/choledochojejunostomy/whipples
Irish women GDm Scenario
Ankylosing spondylitis..symptoms not improving with nsaids what next?
Sulphasalazine?metho?infliximab
6. Hand book pic cerebral tumor
7. Factitious disorder
8. Lots of bad ecg..one of them is bradycardia asking rx
9. Polymyalgia rheumatic scenario..i think..ask inv..all options are weired..no
ESR..
10.Pneumonia x ray rx amoxiclav
11.Aboriginal mom dont want to hold dchild during immunization..no option
for aboriginal health worker
12.Tertiary hospital mi scenario..mx
13.Case control..cant rememberd scenario
14.Liver ct:simple cyst/hydatid/abscess/hemangioma

15.

Pneumothorax scenario

16.Sceneio of upper abdominal pain..hard to differentiated between acute


cholecystis or pancreatitis or PUD..
17.Old woman with greenish brown vaginal discharge..old recall
18.Red face pic..ask inv..ct chest..cant remember others
19.Truck driver with diarr ask inv or dx..cant remember
20.Cleft palate scenario..genetics?carbamazepine
21.Appendicectomy ..after certain days 4 or 5?developed fever cause of
fever asked.
22.A pt irritated drowsy came to u..what will u check first?blood
alcohol/urinary drug screen/breath alcohol
23.Visual hallucination scenario..a wave coming towards hers..stares
blankly..which lobe are affected?medial temp?occipital
24.Pic of neck swelling..no symptoms for yrs..lymph node/lipoma

25.4 yrs old child with palpitation..asking mx?adenosine/immersion of face in


cold water
26.Child fell from tree..ask inv..
27.A scenario of HIV/EBV
28.OLD RECALL OF CAPSULE ENDOSCOPY
29.Achild with varicella planning for a tripcancel trip.?
30.Primary amenoorrohoea scenario..asking dx?mullerian agenesis/ovarian
agenesis..
31.Mile stone at 1 yr another 3 or 4 yrs
32.Cleft palate scenario..mother took carbamazepine during preg..coz?
genetics/carbamzepine
33.Painless hematuria scenario..ct scan/cystoscopy..may be 40 yrd old
man ..already forget
34.A man asking for prostate ca screening..what to do?PSA &DRE
Best of luck guys..lots of new q..very very bad ecg..not standard ecg like blue
book..questions are seems to be confusing..but dont be panic..stay cool and
answer them with confidence..recalls are help to save time..study blue
book very well & study recalls as much as possible..u will pass surely.GOD
BLESS US ALL..

1. The concern about mammography screening in AustraliaA. Missing small cancers


B. Radiation exposure
C. Mastalgia
Ans: Apreviously answered in this month
2. Truck driver with abdominal cramping pain, bloating,constipation and sometimes
diarrhoea. Colonoscopy 12months back was normal.......don't remeber the full stem
Next best mx?
A. Repeat colonoscopy
B. Stool for ova and cyst
C. Psychotherapy
Ans: B

Saifun Nahar IBS is a diagnosis of exclusion. A thorough physical


examination, investigations (FBE, ESR and stool
microscopy or culture) and sigmoidoscopy are necessary

The stem did't have a single word suggesting giardiasis such as watery/offensive/frothy etc
and the options didn't have ssri/tca
3.Sheep farmer with upper rt.Quadrant pain sometime back then resolved prsented with a
CT scan and on p/e temp 37*. ........
A. Hydatid cyst
B. Liver abscess
C. Heparocellular carcinoma
Ans: Mahwish Sheikh depends on CT dear...37 is normal temperature....might be hydatid cyst..

4. Fundoscopy of CRAO presented after 30 min. Immediate mx?


A.Global massage
B. IV acetazolamide
Ans:b
5. Young man in ED after MVA, alert, remembers the event, talking but getting breathless.
Pulse 110/m, bp- 85/? , chest dull on purcussion on rt side, trachea shifted on the same
side......yes Irfan Ahmed' s recall....what to do next?
A. Needle thoracostomy
B. Chest tube
A good friend explained after the exam that it can be hemopneumothorax with collapsed
lung. Thus trachea' s shift on the same side! Very logical!
Ans: B..it could be haemopneumothorax but anyway hemothorax , hemopneumothorax
or pleural effusion chest tube is the treatment of choice.
6. One q about capsule endoscopy!
7. The typical recall about the child presenting in ED with stridor, cyanosis...nebulised
adrenalin! i think it mentions brought in ambulance which means already got O2
8. GDM screening 2 questions- everything same but one with risk factor and the other
without.
9. CTG given with case as follows- primi at term labour ruptured membrane liquor stained.
FHR-140. What to do next?
A. C/S
B. Fetal scalp sampling
C. Continuous CTG monitoring
Ans: previously answered this month
10. 2 questions from fibromyalgia rheumatica...one about confirming diagnostic test and
one
about dx.
Ans: ESRbiobsy
11. One pic with a young woman typical malar rash and some symptoms mentioned asked
for diagnostic inv?
A. ANA
B. Anti smith antibody
Ans:B
12. A man in a metro hospital after MVA arousable, incoherent speech, withdraws to pain,
rt sided scalp hematoma, rt sided pupil dialated, consensual reflex present on lt but absent
on rt....scenario of rt extradural hematoma. Next mx?
A. CT scan head
B. Rt Burrhole
C. Lt. Burrhole
Ans: A
Emad Askary A. Indications for emergency Burr Hole Craniotomy: Patient with GCS<8 with imaging evidence
of an extra-dural haematoma causing midline shift and unequal pupils WHEN TIMELY NEUROSURGICAL
INTERVENTION IS NOT POSSIBLE. Attempts should always be made to discuss the images and necessity for a

procedure with a neurosurgeon. Source: http://www.sjtrem.com/content/20/1/24/ Moreover, he is in a central


hospital

13. Old lady on maintenance dialysis genrally liked by the renal ward nurses. SSometimes
they take her out for walk or giver her cards. However, dialysis unit nurses say she is very
rude and abusive. What is the condition in the old lady?
A. Reaction formation
B. Splitting
Ans: B
14. Young man went for the medical for army found to have hematuria. 50?/ 500? RBC in
microscopy. Glomerular in origin? .....Dx?
A. Thin membrane d/s
B. Alport syndrome
Ans: Asee also JM 808
Shilpi Singh IgA has haematuria followed by throat, influenza or URTI...Thin basement membrane disease
(TBMD, also known as benign familial hematuria and thin basement membrane nephropathy) is, along with IgA
nephropathy, the most common cause of asymptomatic hematuria. The only abnormal finding in this disease is
Most patients with thin basement membrane disease are incidentally discovered to have microscopic hematuria
on urinalysis. Its importance lies in the fact that it has a benign prognosis,[1] with patients maintaining a normal
kidney function throughout their lives.
Saifun Nahar Thin basement membrane disease must be differentiated from the other two common causes of
glomerular hematuria, IgA nephropathy and Alport syndrome. The history and presentation are helpful in this
regard:
In Alport syndrome, there is often a family history of kidney failure, which may be associated with hearing
impairment. Also, males tend to be more affected as Alport syndrome it is X-linked in most cases.
In IgA nephropathy, episodes of frank hematuria are more common, and a family history is less common.

15. Pic of frostbite toe saying a man had a snow ski trip a few days back now presenting
with the toe as in pic...the pic was of early frostbite changes better state than the one we
see in the recalls. So i marked review in a few weeks
16. The recall about the old woman being tearful and depressive having similar episode
when she was 23 yrs old and had lost her son....
Forgot the options but were simliar to what we see on goldstars
Answered in this month
17. Cleaner woman came with pain in the forearm and hand/wrist tenderness in the outer
elbow
A. Myositis ossificans ....
B. Stress fracture of the radial head
Ans: none of the above, It is tennis elbow mostlysee JM 667&1355.if tennis elbow is not
n the choics then B.

Reshma Rafi Signs and symptomsEdit


Pain on the outer part of the elbow (lateral epicondyle)
Point tenderness over the lateral epicondylea prominent part of the bone on the outside of the elbow
Pain from gripping and movements of the wrist, especially wrist extension[citation needed] and lifting movements
Pain from activities that use the muscles that extend[citation needed] the wrist (e.g. pouring a container of liquid,
lifting with the palm down, sweeping, especially where wrist movement is required)
Morning stiffness[citation needed]
Symptoms associated with tennis elbow include, but are not limited to: radiating pain from the outside of the
elbow to the forearm and wrist, pain during extension of wrist,[citation needed] weakness of the forearm, a painful
grip while shaking hands or torquing a doorknob, and not being able to hold relatively heavy items in the hand.
The pain is similar to the condition known as golfer's elbow, but the latter occurs at the medial side of the elbow

....

18. the recall of the man in hypomanic state....but there was no lithium in options
A. Citalopram
B. Clozapine
...don't remember others!!!
Ans: JM 484 drugs in order are 1-lithium 2-sodium valproate 3-carbamazepine 4-second
generation anti psychotic e.g.olanazepine.if any of these choices is not available we
have to choose clozapine although it is used in drug resistant schizophrenia JM 483.
19. ECG of first degree heart block asking which of the following drug can cause it?
I marked Digoxin.
20. 2 more ECG related questions- one the typical lightheadedness after running 15km, but
yes even with full options no PSVT like ecg!!! I marked one in the name of God!!!!
The other don' t remember the stem even!!!!
21. One question with lab results hb low, platelets low, rouleaux formation....
A. ALL
B. Multiple myeloma
Don't remember others!
Ans: Not clear scenario may be B
22. 27 yr old with LSIL on pap smear what next?
A. Repeat after 1 yr
B. Repeat after 6months
C.Colposcopy
D. Hysterectomy
Ans: A JM 928&931,932

23. unilateral renal artery stenosis scenario choice of antihypertensive?


A. Peridopril
24.Rt lower chest pain radiating to the upper abdomen......don't remember what was in the
middle...could be something about pneumonia...o/e temp 37*, abdominal guarding
present but no rigidity on tenderness. Dx?
A. Lung abscess
B. Empyema
C. Subphrenic abscess
Ans: most probably C
Noni RJ I think its C Shan Saleheen & Manal Mohammed. According to this sample
case. http://www.ncbi.nlm.nih.gov/pubmed/11487910 and base on Current Diagnosis & Treatment Surgery
Chapter 22 Disease & Disorder of Peritoneum : Clinical Findings
SYMPTOMS AND SIGNS
An intraperitoneal abscess should be suspected in any patient with a predisposing condition. Fever, tachycardia,
and pain may be mild or absent, especially in patients receiving antibiotics.
22 October at 07:54 Like 7

25. A person getting olanzapine had labs showing abnormal FBS, what next?
A. Lipid profile
Other investigations
HOMCQ 3.369 P.612
26. Pic of young man's red and tender scrotum for 2 days, possible organism asked.
A. Chlamydia
B. Gonococcus
C.E.coli
Ans : A
Alaa Aldeen Abusharkh A....Chlamydia trachomatis and Neisseria gonorrhoeae are the most common
organisms responsible for bacterial epididymitis in men under the age of 35 [22,25,26]. Sexually transmitted
organisms are less likely to be the cause of epididymitis in older men, Escherichia. coli, other coliforms, and
Pseudomonas species are more common. UPTODATE

27. Old recall about diabetic foot ulcer, treatment asked.

28. Old recall about stridor after thyroidectomy, what to do?


Remove all sutures in ward.
29. Old recall of female in lithotomy position, which muscle affected.
30. The recall about 95 Confidence interval of .bp
31. Fluid requirement pre appendicetomy in a 20 yr old man

Sorry all goldstars! I think i stuffed up big time!! First thought i didn't get many recalls but
thinking retrospectively, i understand that i actually did! But i became extremely nervous

which took its toll on the exam!! One thing i did wrong was to be too quick to answer
questions! and this is why i don't remember the questions well!!!! THINK BEFORE YOU
CLICK!! I didn't do that!! I feel really guilty for being unable to provide many recalls!
Actually, a lot of ppl i know kept telling me about "time management "! I was sooo afraid of
it that i actually completed all 150 questions 40 min ahead of time! This back fired on me!
This is NOT how it should be! Should have taken a little more time for each question. Ppl
told me u can only review the flagged questions- WRONG!!! At the end of 150 questions, u
get the options of reviewing- ALL question, Only the flagged questions and also only the
UNflagged questions. So in my remaining 40 min. reviewed the first 15 questions and then
the flagged ones only!
My advice to myself and others is to assess your own capabilities rather than comparing
with others! If u ARE a slow reader then yes be a little careful but if u are a comfortable
reader then concentrate on the questions!! if i pass this time it will only be due to God
listening to all your prayers and if i don't, purely my own stupidity!! Next time i'll make it
Inshallah!

Recalls October 16, 2014,Compiled & Reconstructed By Irfan Ahmed (For


Goldstars Friends Only)

1. A 34 year old lady admitted with sudden loss of vision with confusion and decrease
mental state. Her visual acuity decreasing with other vague symptoms. What would be
your next Inv.
a) MRI Brain
b) FBE
c) Urine R/E
d) Lumber puncture
Ans: A..Multiple sclerosis
12. A 55 year old builder came with weakness of his left hand and leg not lasting for 12
minutes, same complaints couple of weeks back. he is taking only Aspirin, no other
drug, what will you advice him other than usual Mx (October 16 recalls)
a) Do not do strenuous exercise
b) Do not drive for 6 months
c) Add warferin
d) Add more anti coagulation
Ans: answer is not included

JM 1333&1334 if we add dipyridamole to asprin it will be more effective but it is not an


option here. Warfarin can be used alone as it is contraindicated to be used with asprin but it
is not an option as well.
13. A mother came with her 12 month old boy, complaints he is not developing well, what
symptom associated with his developmental delay?
a). do not feed with spoon
b) do not understand the word no
c) Cannot walked unassisted
Ans: B
14. A 25 year old lady with32 week pregnancy came with fatigue, jaundice and itching,
AST/ALT/GGT slightly increased, ALP very high, Dx asked
a) Pregnancy Cholestatis
b) Acute Cholecystitis
c) Pancreatitis
Ans: A..Kaplan P.94
Namrah Shafiq itching, 3 trimester, raised ggt and alkphosphatase increase is cholestasis of pregnancy

15. A 55 year old lady presents with fever, RUQ Pain, her sclera slightly yellow, USG
shows multiple gall stones, BP is Low 80/55, what will be next Mx
(Almost 10 questions like this)
a) MRI
b) ERCP decompression
c) Antibiotic
d) Intravenous cholangiogram.
Ans: C
Shan Saleheen Yes dear Zainab Khalil if IV fluid is in option that would be the right answer as BP low. if
not, then "prophylactic antibiotic" is the right option. "Antibiotic prophylaxis is indicated in the setting of
suspected biliary obstruction, known pancreatic pseudocyst, or ductal
leaks."http://www.asge.org/.../123465e3317a42b4a8e4c826ae1b213d.pdf

16. A 66 year old man taking donepegil now presents with depression and prescribed
setralin 100 mg, developed diarrhoea, palpitition what would you do next
a.
b.
c.
d.

cease Setralin
cease donepegil
cease both
d. reduce setralin dose.
Ans: D
Noor Eldeen D!!!The co-administration of once-daily oral donepezil HCl 5 mg for 15 days and
once-daily oral sertraline HCl (50 mg for 5 days increased to 100 mg for 10 days) did not result

in any clinically meaningful pharmacokinetic interactions, and no unexpected AEs were


observed
Noor Eldeen initial dose of sertraline is 50
Shan Saleheen D, Fahmida Rashid dear, patient's presentation here is only side effects of
sertraline because of higher dose , and initial starting dose is 50mg, so, only dose reduction
would be the initial step.
Shan Saleheen And, Serotonin toxicity is best characterised by a triad of clinical effects: 1.
neuromuscular excitationhyperreflexia, clonus (inducible or spontaneous), ocular clonus,
myoclonus, shivering, tremor, hypertonia or rigidity 2. autonomic effectshyperthermia (mild:
less than 38.5 C; severe: greater than 38.5 C or rapidly rising), diaphoresis, flushing,
mydriasis, tachycardia 3. central nervous system (CNS) effectsagitation, anxiety, confusion.

17. ( I was laughing to get this heck of very very old recall ...lol)
A 25 year old man came to AD with a step wound to his chest. He is aware about
everything from the event. He is having breathless and on examination there is
dullness on percussion and trachea is shifted to that side. BP is 80/50. After doing
initial resuscitation and iv fluid what will you do next?
a) needle thoracotomy
b)strapping
c) cut chest
d) under water seal drain
Ans: D
Rumaisa Shaukat d its hemothoax but tube her then d

18. ( My recalls 16th oct)


A nurse has a needle prick injury from a known hiv positive pt. What would be your
nest step of management after sending blood sample.
a) give immunoglobulin
b) commence antiretroviral
c) wait for serology result
d) give both anti retro + immunoglobulin
b) Ans: b

JM1353

19. A 55 year old lady came with RUQ pain which resolves gradually. She doest have
any other symptoms. On CT scan following was found. What is the Dx

(Picture was like this but smaller)


e) simple cyst
f) Hydatid cyst
g) Hepatoma
h) HCC
Ans: A
20. A 28 year old previously healthy young male came after gym with palpitation, light
headedness, His pulse was regular. His ECG was done and found following. Which
will be his rhythm stripe? (I reconstructed the ECG rhythm strips but it actually
looked the same)

a)

b)

c)

d)

e)
Ans : A supraventricular tachycardia

Hina Asher Strip c is heart Block n d is flutter n e is vit


Rumaisa Shaukat b is with pacing

21. A 61 year old man came with chest pain agitation and sweating, after doing ECG
following was found. He was given Aspirin, Morphine and Nitroglycerin. What will
you do next? (ECG Low quality)

f) Coronary Angiogram
g) tPA
h) CKMB
i) IV Heparin
j) SC LMWH
Ans: Arush Arora tpa in rural setting and angoi with PTCA in tertiary setting
22. A patient presents with Streptococcus bovis infective endocarditis on blood culture.
On Echo he was found MR. Treatment was given and patient responded. What will be
your next step?
a) Refer for colonoscopy
b) Gallium scan
c) MRI
Ans: A
Minahil Obaid S bovis...inc risk of colon cancer
Shilpi Singh S. bovis and Clostridium septicum, which are part of the natural flora of the bowel, are
associated with colonic malignancies. When they present as the causative agent in endocarditis, it
usually call for a concomitant colonoscopy due to concerns regarding hematogenous spread of bacteria

from the colon due to the neoplasm breaking down the barrier between the gut lumen and the blood
vessels which drain the bowel

23. A 13 year old girl living outside for 6 months came with an unwanted sexual
relationship last night in a party. What will be your next step?
d) Call the police
e) Inform parents
f) Notify Child rehabilitation center
a) Ans: C.or sexual assault service or Notify child protection authority

24. A 13 year old girl having relationship of her school mate came to you and she doesnt
want to inform her parents? What would be your next step?
b) Inform parents
c) Arrange pap smear
d) Notify child protection authority
Ans:C
25. A child presents with swelling after 2 weeks of URTI. His urine found RBC ++++,
Protein ++ what would be the Dx?
a) IgA
b) APSGN
c) MSCGN
d) Neprhotic
Ans: B
26. ( My recalls 16th oct)
A nurse has a needle prick injury from a known hiv positive pt. What would be your
nest step of management after sending blood sample.
a) give immunoglobulin
b) commence antiretroviral

c) wait for serology result


d) give both anti retro + immunoglobulin
Ans: b

JM1353

27. A pregnant lady came with severe frontal headache and at 36 week. What would be
your next step of Inv?
f) LP
g) MRI
h) FBE
i) Blood Culture
j) Noncontrast CT
Ans : insufficient stem otherwise C to exclude HELLP syndrome
Moslem Shahrdami Stem is insufficient!!!
If we're looking for preeclampsia, it's mostly diagnosed after 20 week except some condition including
mole hydatiform, twin pregnancy and fetal erythroblastosis can present with preeclampsia before 20
weeks!Without any clues how can we approach to the patient! It might be a migraine, cluster or tension
headache attack or SAH or preeclampsia..
Emad Askary Shilpi Singh, C, to rule out HELLP JM5 p.1027 & 1029.

28. A lady came with a swelling of her leg for 4 months. H/O mosquito bite. What would
be your Dx?

d) Fibroma
e) Nodular BCC
f)

Melanoma

g) Keratoacnathoma
Ans: A
JM 1178
29. A 68 year old man undergone cataract surgery and dischange, on 4th day he came with
pain in left eye, redness, with upper eyelid swelling. ( Picture was almost like the
follwong). What would be your Dx?

a) Hypopyon
b) Acute Glucoma
c) Hyphema
Ans: Mostly A
30. A patient came with severe photophobia, redness and pain in the left eye. (Picture was
almost like following) What would be your Dx?

a) Keratitis
b) Conjuctivitis
c) Glucoma
d) Iritis.

Ans: A mostly but may be C if you are convinced that the pupil in the picture is
dilated
Marzuk Rahman Saifun Nahar But Pain, and redness is present in all of them except conjunctivitis,
which is painless
the difference between the 3 is on the pupil size
Keratitis - pupil normal
Glaucoma - pupil fixed & Dialated
Irititis (/Uveitis) - pupil irregular and constricted.
And pupil seems normal in this picture
So i'm guessing it's A?

31. A patient came with a single swelling in the neck (Almost look like the following
picture) in the following picture. What would be your initial inv?

a) Thyroid Scan
b) FNAC
c) USG
d) CT Head & Neck
Ans: B ..Kaplan medicine P.21
32. A 68 year old lady came with a swelling in front of the tragus for 4 months. What
would you do for Dx?
a) CT Head & Neck
b) FNAC
c) Biopsy of Parotid gland
Ans: B
Shan Saleheen Excisional biopsy of parotid tumours risks damage to the facial nerve and seeding of
the wound, and recurrence may develop up to 20 years after the first attempt at resection.

33. A 56 year old man came with arthritis with dry eyes and swelling of the salivary
gland. What would you do for the swelling?
a) CT head & Neck
b) Biopsy
Ans: BSjogren syndrome
34. A patient came with palpitation, breathlessness. He has done ABG and shows PH:
7.54, Po2: 102, Pco2: 28, what would you do next
e) Rebreathing mask
f) O2 high flow
g) Na bi Carb
h) Intubate
Ans: A
Shan Saleheen respiratory alkalosis due to hyperventilation

35. A 25 year old man came after MVA with horeseness of voice, loss of aortic knuckle,
and widening of mediastinum, BP stable.(Same recall)
e) Transcutenous Echo
f) Transthoracic Echo
g) CT angiogram
h) MRI
Ans: C
Moslem Shahrdami Amir Fthi
Based on medscspe:
Medical management remains the treatment of choice for descending aortic dissections unless they
are leaking or ruptured. With the progress in stenting technology, descending dissections can be
approached with this modality in selected cases.
Moslem Shahrdami Amir Fthi
Shilpi Singh
Based on medscape:

Chest radiography is the initial imaging technique and may or may not reveal any abnormality.
Computed tomography (CT) is useful in hemodynamically stable patients; emergency CT
angiography with 3-dimensional reconstruction is rapidly becoming the diagnostic test of choice.
Magnetic resonance imaging (MRI) is as accurate as CT and may benefit patients who have
adverse reactions to the use of intravenous contrast agents. For hemodynamically unstable
patients, echocardiography is ideal.

36. A 55 year old lady presents with fever, RUQ Pain, her sclera slightly yellow, USG
shows multiple gall stones, BP is Low 80/55, what will be next Mx
(Almost 10 questions like this)
a) MRI
b) ERCP decompression
c) Antibiotic
d) Intravenous cholangiogram.
Ans: C
Shan Saleheen Yes dear Zainab Khalil if IV fluid is in option that would be the right answer as BP low. if
not, then "prophylactic antibiotic" is the right option. "Antibiotic prophylaxis is indicated in the setting of
suspected biliary obstruction, known pancreatic pseudocyst, or ductal
leaks."http://www.asge.org/.../123465e3317a42b4a8e4c826ae1b213d.pdf

37. Two q of same stem. A 51year old lady come with jaundice , itching, Fever, and RUQ
pain, USG done with following findings , in one Q CBD was 14 mm another Q CBD
was 9 mm(exactly the same USG on my exam only stones are bigger) . What would
be your next step

a) IV cholangiogram
b) ERCP
c) MRCP
d) PTC

Ans: B
Nabiha Binte Ali normal diameter of the common bile duct is around 4-5mm

38. ( My recalls 16th oct)


A nurse has a needle prick injury from a known HIV positive pt. What would be your
nest step of management after sending blood sample.
a) give immunoglobulin
b) commence antiretroviral
c) wait for serology result
d) give both anti retro + immunoglobulin
Ans: B.JM p.
39. a man came with breathlessness, no other sign, on exam bilateral pulmonary crackles,
chest X ray infiltrate, no other sign what will you give
a) amoxy+ Clav
b) Doxy
c) Penicillin
d) Metro
Ans: B.atypical neumonia
40. A lady 39 week primae pregnancy with 4 hour labour, Head 2+ station, Membrane
intact, Cervix 2 cm, moulding. Which of the following would indicate delivery by CS
a) If scenario do not change in next 4 hour
b) If Cervix 2.5 cm after next 2 hour
c) If membrane rupture cervix 3 cm in next 2 hour
d) If cervix 4 cm membrane intact station+1 position in next 4 hour.
Ans: A
41. (My recalls 16th October)
A GP measured BP of a group of people mean BP was 150 (at 95 % confidence
interval 110-180 range). What does it mean?
a). It is 95% confident the BP range is 110-180
b) 2.5% is above the BP 180

c) it is 95% sure that mean BP is 150


d) rest I forgot
Ans: May be A

42. (My recalls 16th oct)


A 24year old man confined with a friend came mentioning " funny business is going
around" ( same in inverted comma) he said he has been followed and something will
happened with him after his some promotion & award. What is the example of his
comments?
a) prodormal schezo
b) passivity phenomenon
c) idea of reference
d) delusional disorder
Ans: controversy may be D,A or C
JM 481
With D:Namrah Shafiq d Delusional disorder is distinguished from schizophrenia by the presence of
delusions without other symptoms of schizophrenia (see see Schizophrenia). The delusions may be
nonbizarre (ie, they involve situations that could occur, such as being followed, poisoned, infected, loved
at a distance, or deceived by one's spouse or lover) or bizarre (ie, they involve implausible situations
such as believing that someone removed their internal organs without leaving a scar).
In contrast to schizophrenia, delusional disorder is relatively uncommon. Onset generally occurs in
middle or late adult life. Psychosocial functioning is not as impaired as it is in schizophrenia, and
impairments usually arise directly from the delusional belief.
marcks manual
With A: Emad Askary Saifun Nahar, A. Oxford Handbook of Clinical Specialties 8th ed. P.357:How do
schizophrenic patients explain what is happening? Sometimes mental illness is incomprehensible
forever. For others the turmoil of `mental breakdown` is later seen as a mental breakthrough`. Here is a
quote from a man with schizophrenia: I attained a level of transcendence and awareness that I never
had before. My cognitive, writing, speaking, communication, insight and understanding abilities suddenly
reached a level on their own Amir FthiDr-p BnDr-Sarmad SameerEman Khamis

With c: Zainab Khalil c for me Mahwish


Zainab Khalil here the ques is asking about example of his comments not his condition..check wiki or
any other site..theyve mentioned examples for delusion of refernce and idea of reference
Minahil Obaid Idea of reference ...patient can doubt their ideas if they r provided some insight
Delusion of refernce...patient is adamant abt their thoughts.

43. (My recalls 16th oct)


A 25 year old lady came with photophobia, neck stiffness. Csf finding given ,
lymphocyte high, slightly increased neutrophil, protein upper limit of range, glucose
normal. What will you do?
a) iv ceftriaxone
b) iv acyclovir
c) do mri
d) repeat lumber puncture.

Ans: vague options, supportive care should be present as it is viral


meningitis.otherwise it is MRI if we suspect multiple sclerosis

44. A 18 year old lady presents with premenstrual symptoms. What measure will resolve
her symptoms.
a) evening primrose oil
b) ssri
c) cbt
Ans: C.JM 981
Nabiha Binte Ali incomplete stem. If the stem reveals severe symptom then B. otherwise C. But as it's
said Resolve symptom then better to mark B.

45. A GP got 3 cases of salmonella. To find out the cause what method he should follow?
a) asking the food habits of the case.
b) take two group of people and follow their food habit
c) take history of two group of people took same food one group got the infection
other did not
d) take the history of the group who didn't got the infection
Ans: C

Rehenuma Tarannum Lazuly first question C... we have to find for the source of food..not the usual
food habit..food habit are developed from childhood not just before a infection
Rehenuma Tarannum Lazuly i don't know.. i will just stick to the fact that you have to focus on the the
food source in salmonella patient to find out the cause..Rest i am relying on the actual stem

Emad Askary The 1st question: C. case control study can help determine the causal relationships, and
is also useful for studying conditions with very low incidence or prevalence (Kaplan).

46. (Similar question added )


3 patients with Salmonella in your GP setting in 1 week. What study will u do?
A. Cohort
B. Case control
C. Case series

Ans: B
The 2nd question: B. Please keep in mind that according to Kaplan, case series are used to find a list of
criteria for diagnosis the disease through finding common elements shared across the patients.

47. ( I was very happy to get this q because this was the easiest one CAGE questionary)
A alcoholic man came for treatment of intoxication. After successful resuscitation
what question will help you to diagnose his dependency?
a) any morning compulsory drinking
b) he drinks when he is anxious
c) he drinks socially
d) he drinks when he is driving
Ans: A..JM 1219
Anuruddha Ranaweera a.. cage questionaaire

48. A 28 yr old lady got atypical antisphychotics and become obese and DM 2. Whay
screening is important for her at this moment
e) Lipid profile
f) BP
g) Kidney
h) Cardiac

Ans: A

Namrah Shafiq Shan Saleheen can it be fasting glucose measurement as jm pg 188


Shan Saleheen yes, as here already mentioned that he has DM2 so i think nextg would be lipid profile. If
nothing else was mentioned then we should do FBS first,

49. A lady at 12 weeks gestation came for 1st antenatal check up has family
history of DM2 what you advice her?
a) FBS now
b) 75 mg GTT now
c) 50 mg GTT now
d) 75 mg GTT at 18-20 week
e) 50 mg GTT at 18 20 week
Ans: BJM p.

50. A man after some heavy religions ritual develops excessive hand washing during the
opening of door knob. What would you do as behavioural modification?
a) Insight oriented therapy
b) Group therapy
c) Problem solving
Ans : A.obsessive compulsive disorder.
Amgad Samir Alwayas for OCD, it's exposure and response prevention, desensitization or
deflooding. But if no other option go with insight oriented

51. A man came with sudden loss of visual acuity on his left eye. The no carotid bruit.
But fundoscopy shows following findings. What would you do?

(Image was quiet like this but the other eye)

a) IV acetazolamide
b) Pilocarpin
c) Eye massage
Ans: A

Vous aimerez peut-être aussi